Praxis Practice Questions

¡Supera tus tareas y exámenes ahora con Quizwiz!

A 2-year-old named Spencer was referred to your office because other professionals have suspected a potential speech and language disorder. Which of the following characteristics that Spencer exhibited during your assessment does not seem right for his age? A. He rarely used initial consonants and when he did, they were often misarticulated. B. Spencer was able to understand most things that you said to him as he answered questions with the correct responses. C. About 2/3 of Spencer's speech was intelligible. D. Spencer walked with characteristic toddler movements and was almost always on tiptoes.

A

A 2-year-old with an unrepaired cleft palate was recently seen for an evaluation. When comparing the child's data to developmental norms, the speech-language pathologist determines that the child has not mastered bilabial stops. Why would developmental norms not determine whether the child receives speech treatment? A. The child does not have the anatomical capacity to produce bilabial stops. B. The speech-language pathologist would need to consult with medical professionals before determining treatment. C. Developmental data do not apply to every child. D. Every child develops at a different pace.

A

A 45-year-old man was referred to a gastrointestinal physician for an evaluation because of respiratory distress and difficulties swallowing. The physician conducted a barium esophagram study and noted an oblique filling defect in the posterior esophagus. On completing her evaluation, the attending cardiologist also mentioned to the patient that she noticed an aortic arch anomaly (an aberrant right subclavian artery-the right subclavian artery arose from the left side of the aortic arch and was externally compressing the esophagus). The cardiologist mentioned to the patient that this vascular anomaly in the thorax, particularly the aortic arch, was compressing the trachea and esophagus and resulted in the patient experiencing significant respiratory distress and feeding difficulties. This condition is called A. dysphagia lusoria B. stenosis of the right subclavian artery C. choanal atresia D. cyanosis

A

A 60-year-old patient fell and hit his head on a bathtub, sustaining a traumatic brain injury (TBI). You have physician orders to evaluate his responsiveness and communication. When you screen this patient, he is generally unresponsive to verbal or tactile stimuli. The following assessments may be an appropriate choice for a full evaluation of this patient, except: A. The Galveston Orientation and Amnesia Test B. The Rancho Los Amigos Levels of Cognitive Function C. The Glasgow Coma Scale D. The Disability Rating Scale

A

A 72-year-old patient has been depressed since her husband passed away 2 months ago. The onset of her symptoms was short and abrupt. When memory was tested, she often stated that she did not know the answer to a question. Her attention and concentration appeared to be intact; however, she seemed upset and distressed during the evaluation. After extensive testing, the psychiatrist diagnosed the patient with: A. Pseudodementia B. Dementia C. Pick's disease D. Alzheimer's disease

A

A clinician has been asked to evaluate a patient, Sydney T., who is 65 years old. He was rushed to the emergency room because of trouble with speaking, numbness and loss of movement in his face, vision problems, and confusion understanding simple statements. After completing a medical test, it was determined that Sydney had suffered a cerebrovascular accident (CVA). According to the medical chart, he "has trouble swallowing and suffers from speech and language problems secondary to the CVA." The dietitian is concerned that Sydney has lost weight and is not eating all his meals. The report indicates that "the patient seems depressed and uninterested in eating." The nursing staff indicates that he eats minimally during breakfast, lunch, and dinner. Reportedly, the patient seems to be unable to produce intelligible speech, and the certified nursing assistants are unable to understand him. Sydney's wife indicated that prior to his hospitalization he was an outgoing person and coached high school football. As previously stated, Sydney has trouble swallowing. During the modified barium swallowing study, the clinician also noted that he has a delay in the pharyngeal stage of swallowing. The clinician observed that he had a 5-second delay before the pharyngeal swallow was triggered. The clinician noted in the chart that the average duration of a normal pharyngeal swallow is A. 1 second. B. 2 seconds. C. 3 seconds. D. 4 seconds

A

A clinician in a private practice is approached by the parents of Cole, a 5-year-old boy. They want to place Cole in kindergarten in the fall, but they share, "We know there's something wrong with him—we're just not sure what." According to Cole's parents, he is a "sweet, lovable boy who will go to anybody. He likes to sing a lot, too." Because the parents live in a rural area, health-care access has been limited. After seeing Cole for the first time, the clinician refers his parents to a neurologist because she suspects that Cole has a syndrome. Cole is small for his age and has an elfin-like appearance characterized by a small chin, turned-up nose, puffiness around the eyes, a long upper lip, and a wide mouth. Cole's teeth are small and widely spaced. The clinician explained to her student intern that people with this syndrome A. have a "cocktail party"-type personality. B. have above average height and weight. C. can live independently. D. have normal language abilities as they age

A

A clinician in a private practice is approached by the parents of Cole, a 5-year-old boy. They want to place Cole in kindergarten in the fall, but they share, "We know there's something wrong with him—we're just not sure what." According to Cole's parents, he is a "sweet, lovable boy who will go to anybody. He likes to sing a lot, too." Because the parents live in a rural area, health-care access has been limited. After seeing Cole for the first time, the clinician refers his parents to a neurologist because she suspects that Cole has a syndrome. Cole is small for his age and has an elfin-like appearance characterized by a small chin, turned-up nose, puffiness around the eyes, a long upper lip, and a wide mouth. Cole's teeth are small and widely spaced. This syndrome is caused by a rare genetic disorder that affects an estimated 1 out of every 20,000 babies. It is caused by what? A. An abnormality on chromosome 7, including a gene that makes the protein elastin B. Autosomal dominant inheritance and deletion in the region of the long arm of chromosome 15 (15q11-15q13) C. A spontaneous autosomal dominant mutation whose gene and locus is FGR2 at 10q25-26 D. An expanded number of cytosine-guanine-guanine (CGG) nucleic acid repeats on a specific gene on one of the distal ends of the X chromosome

A

A clinician in private practice receives many referrals from local public school clinicians. The children being referred are those whose language test scores are too high to legally qualify them for therapy in the public schools. The parents of these children want services and are willing and able to pay for them out of pocket. The clinician in private practice often administers the Test of Word Intentionality, Temporality, Tenses, and Rules (TWITTER). One day, she becomes curious about the "typical score" of the children to whom she administers the TWITTER. The clinician takes the TWITTER scores of 21 children and lines them up in order from highest to lowest score: 33 46 48 51 55 60 69 73 73 73 73 82 85 89 91 93 95 95 95 96 98 The clinician concludes that the "typical" score (the one that occurs most often) on the TWITTER is 73. In terms of measures of central tendency, what is the typical score calculated by the clinician? A. Mode B. Median C. Standard deviation D. Mean

A

A clinician is evaluating a 12-year old boy, Justin, who was in a car accident and sustained a traumatic brain injury. He is now having difficulty communicating and has complex communication needs. The clinician is attempting to determine which type of alternative/augmentative communication (AAC) technology to use with him. In evaluating Justin's willingness to use this technology, she must consider response efficiency, which involves: A. quality, rate, and immediacy of reinforcement as well as response effort B. the speed at which Justin can press keys on the AAC device C. whether or not a device's symbols are transparent D. whether or not a device's symbols are opaque

A

A clinician measured the disfluency rates of a client from a speech sample. She then asked another clinician to measure disfluencies using the same method she had used. The first clinician calculated a reliability index based on her measure and that of the second clinician. This index is a measure of A. interjudge reliability. B. intrajudge reliability. C. test-retest reliability. D. split-half reliability.

A

A code is a system of rules for arranging arbitrary symbols in an orderly, predictable, systematic manner that allows anyone to know the code to interpret the message. Which type of code is a speech-language pathologist likely to teach someone with a communication device? A. Morse code B. Steganography C. ROT1 D. Transposition

A

A patient underwent a partial laryngectomy as a form of management for laryngeal cancer. You are explaining to a graduate student clinician that the area surrounding the larynx, including the pharygoesophageal junction, right and left pyriform sinuses, lateral and posterior hypo pharyngeal walls, and the post-cricoid region, is known as the: A. hypopharynx B. supraglottis C. subglottis D. pharynx

A

A patient was referred for therapy to an outpatient rehabilitation facility because of a diagnosis of apraxia of speech post CVA. The clinician decided to use a shaping technique that focused on orofacial and articulatory postures with specific instructions about how to change current speech and non-speech movements to achieve the target sounds. This is an example of A. phonetic derivation. B. phonemic awareness. C. rate control. D. integral practice.

A

A researcher is describing the speech of a group of children who have been diagnosed as clutterers. She finds that the faster the children speak, the less intelligible they are. The researcher obtains a Pearson r correlational relationship of -.89. This shows that there is ___________________ between rate of speech and intelligibility. A. a strong negative correlational (or inverse) relationship B. a positive correlational relationship C. a moderate cause-and-effect relationship D. virtually no correlational relationship

A

A special educator tells a disruptive boy in her class that he cannot have tokens (which can later be exchanged for a small gift) if he leaves his chair and wanders around the classroom. He is reinforced for many acceptable behaviors. This is an example of A. differential reinforcement of other behavior. B. differential reinforcement of incompatible behavior. C. negative reinforcement. D. punishment.

A

A specialist uses a bright light source and a small, round, 21-25-mm mirror angled on a long slender handle to lift the velum and press gently against her patient's posterior pharyngeal wall. Next, the specialist maneuvers the mirror to view the laryngeal structures during quiet respiration and while the patient is producing "eeee." This procedure is known as A. indirect laryngoscopy. B. direct laryngoscopy. C. endoscopy. D. videostroboscopy.

A

A speech-language pathologist who worked in a neonatal intensive care unit treated many children with developmental issues. She asked her student intern what the difference was between reflexive cries and vegetative sounds. How should the student respond? A. Vegetative sounds are associated with feeding while reflexive vocalizations are automatic responses about the physical state of the infant. B. Vegetative sounds are about the physical state of the infant while reflexive sounds are associated with feeding. C. Vegetative sounds are cries, coughs, burps, and grunts while reflexive vocalizations are grunts, sighs, and clicks. D. Vegetative sounds are produced in comfortable states and reflexive sounds are produced in uncomfortable states.

A

A student is reviewing information for a final exam. She discusses in her study group that a cause of cerebral palsy involves _________: A. Injury to the cerebral oxygen levels during the prenatal or perinatal period B. Reduced glucose levels C. Injury to the limbic system D. Damage to the meninges

A

A teacher directs a child to describe the actions of a story that is presented on their communication device via a picture sequence. What is this type of teaching called? A. Structured teaching B. Situational teaching C. Generalization teaching D. Integrated teaching

A

A test of language skills for children demonstrates ____________ when it reflects the child's actual, daily environment and life experience. A. ecological validity B. construct validity C. intrajudge reliability D. standardization adequacy

A

Acoustic immittance is measure with A. tympanometry B. immitanceometry C. impedanometry D. acoustic reflex

A

Acoustical, mechanical, or electrical resistance to motion or sound transmission is called A. impedance. B. admittance. C. immitance. D. velocity

A

An 81-year-old bilingual man from Thailand has had a stroke, and you are seeing him for therapy. He is recovering both his primary language and his English skills, but you are working only in English. No interpreters are available, unfortunately, and the family has indicated that they would prefer treatment to be conducted in English, anyway, because many of the patient's grandchildren speak English fluently. Which one of the following productions would be an example, on the patient's part, of English influenced by his primary language of Thailand and not the stroke? A. "They going over there today." B. "I done got to get dressed now." C. "She not have no money in the bank." D. "We be havin' most fun."

A

An 84-year-old woman with dementia who recently suffered a stroke was admitted to a nursing home, as she had difficulty swallowing. The results of a videofluroscopy study indicated that she had severe pooling in the vallecula and pyriform sinuses. She was also at risk for aspiration. The speech-language pathologist decided to administer syringe feedings. Ethically, the clinician A. should have a rationale for using syringe feedings B. should not worry, as there is a limited chance of feeding the patient to quickly C. should not worry, because with syringe feedings it is difficult to introduce a large bolus with each swallow D. should not consider the patient's cognitive status, since she will not be aware of the syringe feedings anyways

A

An audiologist presents a passage of connected speech to your patient. An excerpt from the passage is as follows: "I went to the beach last week. My mom and I had a lot of fun. We built a sandcastle and collected a bunch of seashells. At the end of the day, we got vanilla ice cream on the boardwalk." Your patient repeats the message syllable for syllable. The material that was selected was within your patient's language level and the passage was interesting to the patient as well. This technique can be used as a tool in aural rehabilitation and it has great face validity because the material approximates normal communication more closely than single words. It can also help to improve listening and spoken language skills. What is this technique called? A. Speech tracking B. Listening training C. Pure-tone testing D. Acoustic highlighting

A

An individual who sustained a traumatic brain injury, presented with articulatory and phonological deficits. In this case, the injury was probably sustained in all of the following, except: A. Hypothalamus B. Cerebellum C. Peripheral Nerves D. Brainstem

A

An investigator carries out a study to answer the question of whether an increased rate of sibling speech causes an increase in the frequency of stuttering in children. After pretesting rates of stuttering in selected children and the speech rate of their siblings, the investigator tells the siblings in the control group to speak as they normally would at home. She tells the siblings in the experimental group to speak much more rapidly than they would at home. In this study, what is the dependent variable? A. Rates of stuttering in the children in both groups B. The rate of speech of the siblings in the experimental group C. The rate of speech of the siblings in the control group D. The combined amount of stuttering by the children in both the experimental and the control groups

A

An outpatient was seen for therapy for acquired apraxia of speech. The clinician instructed the patient to produce speech at one syllable per beat. The beat was set at a slower rate than the patient's actual speaking rate. Hand-tapping was also used simultaneously with this treatment technique. This is an example of A. metronomic pacing B. metrical pacing C. rate control D. integral practice

A

Babies with cleft palate show no difference in which aspect of their language development in comparison with their peers: A. Vocalization frequency B. Canonical babbling C. Consonant inventories D. Cry behaviors

A

Billy is a 16-year-old who recently suffered a traumatic brain injury as a result of a car accident. Billy remained in a coma for several days, and recently became minimally alert. During the early stages of treatment, the speech-language pathologist should focus on: A. Getting Billy to respond to sound, touch, or smell B. Helping Billy understand what day it is C. Finding ways to improve Billy's memory D. Working on Billy's social skills

A

Clients with a cleft palate will almost universally have issues related to which of the following? A. Persistent otitis media leading to conductive hearing loss B. Anatomical deformities causing sensorineural hearing loss C. Otitis media leading to sensorineural hearing loss D. Congenital conductive hearing loss

A

Direct management of communication and memory skills in treating patients with Alzheimer's disease can be useful until the: A. the late stage of the disease B. early stage of the disease C. middle stage of the disease D. patient get irritated with the treatment program

A

Disorders of the oral phase of a swallow are mainly due to A. an anterior, as opposed to a posterior, tongue movement. B. slippage of food into anterior and lateral sulcus. C. a reduced range of lateral mandibular movement. D. a difficulty in holding the bolus.

A

During an evaluation, a young client presented with speech sound errors when producing various multisyllabic words. She made the following errors when saying "elephant" and "ask": /ɛfələnt/ for /ɛləfənt/ and /æks/ for /æsk/. What is the phonological process diagnosis? A. Metathesis B. Epenthesis C. Cluster reduction D. Weak syllable deletion

A

Dylan, a 3-year-old, comes to your office for an evaluation. You review normative data to determine whether Dylan is developing within normal limits. All of the following statements are reasons you should not over-rely on developmental norms except: A. Norms do not accurately tell us how well a child's sounds are developing. B. A norm is only an average age at which a behavior occurs and refers to a hypothetical child. C. True norms are collected from and apply to a normal, randomly selected sample—these samples rarely exist in the real world. D. Different norms are rarely in agreement with each other due to factors such as where or when the study was conducted.

A

External validity of a study may be threatened by A. the Hawthorne effect. B. statistical regression. C. maturation. D. attrition.

A

For children with cerebral palsy who are over the age of 4, motor speech impairments can be measured by using the: A. Viking Speech Scale B. Rancho Los Amigos Scale C. Blood's Fluency Assessment Scale D. Melodic Intonation Scale

A

From the list below, Scaler Scott and Ward (2013) have suggested that ___________ alone may increase intelligibility and speech fluency for those who clutter. A. increased pausing B. taking a breath C. easy onsets D. continuous phonation

A

Grayson, a 55-year-old restaurant owner from Canada, suffered a lesion because of a stroke he suffered at work 2 months ago. Ever since the stroke, he has severe difficulties with naming common objects, along with a slow rate of speech and limited word output during conversation. During his initial evaluation, it was noted that Grayson mainly produces content words, which allows some meaning to be translated during conversation; however, his speech sounds very agrammatical. He also showed difficulties with repetition when asked to repeat back a sentence that the clinician produced. Because of Grayson's acquired deficits, he has been unable to work at his restaurant; however, he wished to return to his beloved restaurant as possible. Grayson receives speech therapy at his home three times a week to address the speech and language deficits caused by his stroke. During therapy, his clinician works on increasing his length of utterances and the complexity of his responses. The clinician also works on decreasing grammatical errors and naming difficulties. Grayson's clinician reports that he appears to get frustrated during conversation, indicating that he is aware of his deficits. Based on the given deficits and treatment description, what is Grayson's diagnosis? A. Broca's aphasia B. Transcortical motor aphasia C. Global aphasia D. Primary progressive aphasia

A

In Appleton City, a speech-language pathologist has a thriving private practice composed heavily of non-native speakers of English who refer themselves to him for accent training. This clinician, Joaquin, works to help his clients increase their intelligibility in English for business purposes. One way that Joaquin evaluates the success of the accent training he provides for these clients is to rate their overall intelligibility of speech before they start accent training and then again after 12 weeks of training. He finds that the clients appreciate these before-and-after measures of their progress. After providing accent training for a number of clients, Joaquin realizes that a potential problem with his before-and-after intelligibility ratings is that he has become accustomed to the clients' speech as he has gotten to know them; this could be affecting the "after" intelligibility rating. Joaquin goes to a local university and selects three speech-language pathology graduate students to watch before-and-after videos of his accent clients and independently rate each client's intelligibility. He finds to his dismay that the three students vary greatly in their ratings of the same clients. For example, one student rates Dr. Osiychuk, a Ukrainian client, as 90% intelligible after 12 weeks of accent training. A second student rates Dr. Osiychuk as only 65% intelligible after 12 weeks of accent training, and the third student rates Dr. Osiychuk as 40% intelligible after 12 weeks of accent training. In this situation, what is an appropriate conclusion? A. There is low interjudge reliability. B. There is high interjudge reliability. C. There is low intrajudge reliability. D. There is high intrajudge reliability.

A

In research, the ___________ hypothesis states that two variables are not related. A. null B. alternative C. correlational D. coefficient

A

In the case of vocal fold paralysis, the following treatment techniques can be trialed in voice therapy to achieve firmer vocal fold closure: A. Hard glottal attacks B. Elevation in pitch C. Decreased loudness D. Head turning or positioning

A

Interaction between caregiver and infant strengthens attachment. When speaking to the caregiver before therapy, it is important to establish the eye contact and eye gaze exchange between the caregiver and the infant. How many weeks does it take on average for an infant to show preference for the eye and the border of the face? A. 3-11 weeks B. 5-6 weeks C. 8-11 weeks D. 9-12 weeks

A

Johnny, a 23-year-old male, was riding his motorcycle at 60 mph on a highway. A truck pulled into his lane, and to avoid hitting the truck, he had to brake and swerve. Johnny lost control and was thrown from the motorcycle. He flipped upside down and struck the railing alongside the road. Johnny was wearing a helmet; however, he hit his upper back and the back of his head when he hit the guardrail. What would be the nature of his head injury be? A. Translational trauma (linear acceleration) B. Angular acceleration injury C. Traumatic hemorrhage D. Impression trauma (nonacceleration)

A

Language and communication impairments of individuals with traumatic brain injuries are often due to impairments in basic cognitive skills, including attention, memory, and executive functioning. These various impairments are most effectively and efficiently treated through _________. A. Direct treatment targeting social and interpersonal (pragmatic) aspects of communication B. Indirect treatment targeting visual processing C. Pharmacologic intervention D. Sensory stimulation treatment

A

Language disorders are often identified through multidisciplinary assessment. A speech-language pathologist may perform a comprehensive language evaluation that includes interviews, observations, questionnaires, surveys, and formal tests. There are two types of standardized tests that a clinician may use during an assessment: norm-referenced and criterion-referenced tests. Which of the following statements is true about criterion-referenced tests? A. Criterion-referenced tests identify what a client can and cannot do compared to a performance standard. B. Criterion-referenced tests are rarely used to assess clients for fluency and voice disorders. C. Criterion-referenced tests allow a comparison of an individual's performance to the performance of a large group. D. Results of a criterion-referenced test are typically reported as a percentile ranking.

A

Large lesions involving more than one region of the larynx usually require the following: A. total laryngectomy B. partial laryngectomy C. supraglottic laryngectomy D. medial laryngectomy

A

Larissa, a 9-year-old client with cerebral palsy, attends speech therapy once a week at a private practice. Larissa has excessive drooling, imprecise articulation, as well as problems with speech rate, intelligibility, resonance, breathiness, and swallowing. The speech-language pathologist mentions to Larissa's parents that the ultimate measure of speech effectiveness for a client with cerebral palsy is to: A. Improve intelligibility B. Reduce drooling C. Reduce breathiness D. Increase speech rate

A

Laryngeal cancers that form on the vocal folds are often detected early because A. the mass interrupts the vibration of the vocal folds causing hoarseness B. of chronic and excessive coughing C. of severe dysphagia D. the mass on the vocal folds results in ability to inhale and exhale

A

Lee, a 42-year-old male, has come to an outpatient center to be evaluated by a speech-language pathologist due to concerns expressed by his wife and children regarding his speech. Upon arriving at the clinic, the clinician notices that Lee's oral and facial structures appear normal at rest. During the evaluation, Lee admits that for the past 5 years, he has noticed moments of random slurred speech during conversation; however, he never wanted to bring up the issue with his family because he was too embarrassed of the occasional drunken sounding speech. He told the clinician, "I just hoped that no one would ever notice." His wife reported that within the past year his conversational speech has been characterized by irregular articulatory breakdowns, reduced rate of speech, and unsteady vowel prolongations. She expressed concerns for her husband's ability to communicate with others who are unaware of his deficits. Lee complained that he is unable to coordinate his breathing with speaking, and often bites his cheek or tongue when speaking. This creates discomfort and frustration for him, but his wife reported that when Lee slows down his speech, he usually improves his intelligibility. According to his family, Lee often seems discouraged that his speech will never improve and he will suffer from the deficits for the rest of his life. During the evaluation, the clinician notices that his wife and three children provide a strong support system for Lee. The first thing the clinician should do when initially evaluating Lee's speech is to: A. ensure that Lee possesses some form of functional communication. B. use strengthening exercises to address weakness. C. teach Lee facial relaxation techniques to use at home. D. educate Lee on the Lee Silverman Voice Treatment strategy.

A

Magdalena, a 10-year-old Austrian student, recently moved to your school. She was diagnosed by the speech-language pathologist at her previous school with selective mutism. You have never treated a client with selective mutism. With all new diagnoses, it is important to educate yourself on the general treatment options. Her parents report that Magdalena does speak at home when the immediate family is around. Additionally, once an unfamiliar person is present, Magdalena seems very tense and anxious. She was able to warm up to the speech-language pathologist from the previous school within a month of therapy; however, she received therapy twice a week. You decide to look up strategies that will be the best fit for Magdalena. Which of the following strategies is most appropriate for her? A. Exposure-based practice B. Systematic desensitization C. Stimulus fading D. Contingency management, positive reinforcement, and shaping

A

Many individuals with developmental disabilities will require augmentative and alternative communication (AAC) devices to communicate. What kind of instruction will clinicians need to provide that includes the use of techniques such as shaping, fading, prompting, and expectant time delay? A. Explicit B. Implicit C. Tactile-kinesthetic D. PROMPT

A

Marisa's mother takes her to the emergency room because she notices a foul-smelling discharge coming from Marisa's ear. The doctor examines Marisa's ear and sees that her tympanic membrane is permanently ruptured. The doctor explains to Marisa's mother that he will need to repair the tympanic membrane by using which of the following surgical procedures? A. Myringoplasty B. Myringotomy C. Middle ear fusion D. Microtia

A

Patients have the right to change their own protected health information (PHI). Under what circumstance would a health-care provider deny a patient's request to amend his or her PHI? A. If the provider already created an accurate and complete document B. If the document is already part of the set of records C. If providers created themselves without any other contributor D. If the record was previously inspected by a governing agency

A

Patients who have undergone total laryngectomy are not in danger of: A. Aspiration of food or liquid during swallow B. Pseudoepiglottis C. Lesions in pyriform sinus D. Swallowing difficulties

A

Periodic waveforms can be thought of as a series of sinusoids. The difference between waveforms and spectrums is the measurements on the y axis and the x axis. What two measurements are needed for the x axis and y axis in a waveform? A. Amplitude and phase B. Amplitude and time C. Amplitude and frequency D. Time and phase

A

Preston, a 58-year old mechanic was diagnosed with aphasia. To improve brain reorganization, the clinician was recommending therapy, based on a theory that includes auditory processing for all aspects of language ability. Therapy would feature varying the load while providing intensive auditory stimulation to target word retrieval, word repetition, sentence formulation, writing, and reading. Which theory is this? A. Schuell's (1964) Stimulation Approach B. Localization: Boston School C. Neurolinguistics D. Cognitive Neuropsychological

A

Priscilla, a 15-year-old patient with a diagnosis of velocardiofacial syndrome, is seen in an outpatient facility. A clinician is currently seeing this patient for language and articulation deficits; however, she notices increased coughing and laryngeal pumping as Priscilla was having a snack. Priscilla's mother noted that this was common, but that she did not think that is was an issue. Further evaluation is recommended, but it is possible that Priscilla has: A. Pharyngeal hypotonia B. Nasal regurgitation C. Oral apraxia D. Decreased anterior to posterior transit time

A

Sam, a 4-year-old boy, recently started to use an augmentative and alternative communication (AAC) device to aid in his communication with others. Although he uses an AAC device, his cognition is within normal limits. Which of the following is true about Sam's language ability? A. He may be delayed in expressive language because of the AAC supports he has been given. B. He should not have the same receptive language as a typically developing child. C. He should not be following the same communication milestones as a typically developing child. D. He will likely have difficulties with phonology and syntax.

A

Secondary consequences of traumatic brain injury are the brain's physiologic response to trauma and can often be more devastating than primary consequences. Which of the following is an example of a secondary consequence of TBI? A. Cerebral swelling B. Impression trauma C. Skull fracture D. Coup ("impact") injuries

A

Select the statement that applies to ethnographic studies. A. They are mostly descriptive B. They are suitable for evaluating treatment effects C. They are well suited for evaluating correlations between variables D. They are not appropriate for studying patterns that are present in various cultures

A

Select the statement that applies to the normal distribution. A. It is based on the arithmetic mean of scores or values. B. It is based on the statistical mode of the scores C. In one, 14.13% of the scores fall within one standard deviation below the mean. D. The 50th percentile is equivalent to the mode.

A

Select the true statement or statements of Van Riper's approach to stuttering treatment. A. its goal is fluent stuttering B. it seeks to establish normal-sounding fluency by slowing down the speech rate C. it seeks to reduce stuttering by using operant contingencies D. it does not involve counseling

A

Sometimes specialists assess the lung volume of voice patients because breath support is inadequate. Specialists can measure _______, or the total volume of air in the lungs; other measurements can include _______, or the amount of air inhaled and exhaled during a normal breathing cycle; and _______, or the volume of air that the patient can exhale after a maximal inhalation. A. total lung capacity, tidal volume, vital capacity B. vital capacity, tidal capacity, total lung volume C. vital capacity, total lung capacity, tidal volume D. tidal volume, total lung capacity, vital volume

A

Spectrums include frequencies that need to be attenuated as well as frequencies that are allowed to pass through a filter without being dampened. Which of these is not a type of filter used for spectrums? A. Mid-pass filter B. Low-pass filter C. High-pass filter D. Band-rejection filter

A

Speech reception threshold (SRT) is the intensity at which an individual can identify simple speech material about 50% of the time. During a basic hearing evaluation, an audiologist is measuring a patient's SRT. The audiologist says words such as baseball, ice cream, and sunset. She then asks her patient to say the words that were uttered. The test materials used by this audiologist are best known as: A. Spondee words B. Monosyllabic words C. Functional words with two syllables D. Multisyllabic words with no stress

A

Speech-language samples are very useful in the assessment of a client's communicative abilities, along with possible disorders that he or she may demonstrate. To obtain both a reliable and valid speech-language sample, you should do all of the following except: A. Strive for a long sample in one environment where the client is most comfortable. B. Vary the subject matter of the sample and preselect topics of interest to the client. C. Establish a positive relationship before collecting a sample. D. Consider the client's age and background to avoid asking questions that could be considered demeaning.

A

The Health Insurance Portability and Accountability Act (HIPAA) includes which area(s) of law? A. Insurance portability, antifraud enforcement, and administrative universalization processes B. Insurance portability and administrative universalization processes C. Antifraud enforcement D. Antifraud enforcement and administrative universalization processes

A

The Joint Commission on Accreditation of Health Care Organizations (JCAHO) uses tracers methodology as a primary way to evaluate if organizations are complying with standards. What are the two types of tracers? A. Patient care tracers and patient system tracers B. Primary care tracers and patient system tracers C. Patient safety tracers and patient system tracers D. Patient care tracers and management system tracers

A

The following syndrome is a form of aphasia that causes a gradual or sudden loss of receptive and expressive language skills in children between the ages of 3-7: A. Landau-Kleffner syndrome B. Pierre-Robin syndrome C. Moebius syndrome D. Prader-Willi syndrome

A

The perturbation theory states that if a change in cross-sectional area is applied (a perturbation), the acoustic effect depends on proximity to a node or an antinode. Which of the statements below is true about antinodes and nodes concerning formant frequencies? A. Near an antinode the formant frequency lowers; near a node the formant frequency rises. B. Near an antinode the formant frequency stays the same; near a node the formant frequency rises. C. Near an antinode the formant frequency rises; near a node the formant frequency lowers. D. Near an antinode the formant frequency lowers, near a node the formant frequency stays the same.

A

The phonetic placement method is used for articulation treatment by showing the placement of articulators to produce a target speech sound. This method is effective when the client cannot imitate the production after a clinician model. Which of the following is not a characteristic of the phonetic placement method? A. Giving minimal visual feedback B. Describing how the target sound is produced C. Showing the placement of the articulators D. Demonstrating how the sound is produced

A

The systematic reduction in the frequency of stuttering when a short printed passage is repeated aloud numerous times is known as: A. Adaptation effect B. Consistency effect C. Adjacency effect D. Audience size effect

A

The theory that asserts that each child is born with an innate language acquisition device (LAD) is the A. nativist theory of Chomsky B. cognitive theory of Piaget C. behavioral theory of Skinner D. social interactionist theory of Vygotsky

A

There are several different types of cancerous cells that may result in laryngeal cancer. What is the most common laryngeal cancer that develops in flat, scale-like cells lining the pharynx? A. squamous cell carcinoma B. spindle cell carcinoma C. Adenocarcinoma D. lymphoepithelioma

A

To measure public attitudes toward stuttering within a context of a variety of human conditions, a speech-language pathologist would like to administer a reliable and valid survey that has good internal consistency and is translatable to several languages. The clinician would choose the: A. POSHA-S B. PATA C. TOCS D. OASES

A

Unilateral upper motor neuron (UUMN) dysarthria can be caused by unilateral damage to the upper motor neurons. When evaluating a client for this type of dysarthria, it is important to recognize the deviant speech characteristics of a client with a diagnosis of UUMN dysarthria. Which of the following is the deficit that is present in almost all clients? A. Imprecise consonants B. Irregular articulatory breakdowns C. Increased rate in segments D. Excess and equal stress

A

Various classification systems for cerebral palsy have been proposed. Six common symptom areas include athetosis, cerebellar ataxia, rigidity, tremors, flaccidity, and __________________. A. Spasticity B. Hypernasality C. Parkinsonism D. Tardive dyskinesia

A

Vibration consists of movement or displacement in more than just one direction. What is the most fundamental form of vibration? A. Simple harmonic motion B. Pure tone C. Sinusoidal motion D. Complex tone

A

Virat, 72-year-old patient, was added to your caseload because he presented with dysphagia. Your evaluation indicated a mild to moderate oral phase dysphagia. During your initial treatment session, Virat noted that he had previously completed oral-motor exercises and that he would like to continue them, as they seemed to help. You decide to trial Virat's stimulability for these exercises as part of his plan of treatment and need to discuss the purpose and goal of using these exercises with him. You explain to Virat that the following is not a goal of oral-motor control exercises: A. Increase movement of the base of the tongue B. Increase range of tongue movements C. Increase buccal tension D. Increase the range of lateral movements of the jaw

A

Vowels are produced with a relatively open vocal tract. There will always be a fundamental and harmonics for vowels. We perceive vowel differences by the relationship between formants. How are vowels perceived? A. Spacing of the formants B. The fundamental frequency C. The height of each formant D. The height of each harmonic

A

What are the two steps of developing contrastive stress? A. Maintaining a single prosodic element and prosodic integration of two words into a one-tone unit B. Maintaining a single prosodic element and prosodic integration of two words into a two-tone unit C. Prosodic integration of two words into a one-tone unit and maintaining a single prosodic element D. Prosodic integration of two words into a two-tone unit and maintaining a single prosodic element

A

What is one difficulty with cross-sectional studies? A. The investigator observes differences between subjects of different ages to generalize about developmental changes that would occur within subjects as they mature. B. The investigator observes differences within subject groups of different ages to generalize about developmental changes that would occur between subjects as they mature. C. The same subjects are studied over time; this is expensive, time consuming, and difficult because subjects might drop out of the study. D. The total age span of children to be studied is divided into several overlapping age spans, and it is difficult to follow subjects from the lower to the upper end of each age span.

A

What is the Positive Everyday Routines approach to intervention that was designed by Ylviasker and Feeney (1998)? A. Provide adults with traumatic brain injury (TBI) with highly trained and relatively automatic strategies to improve their performance in specific areas of their life B. Trains adults with TBI to give themselves enough time to perform daily life tasks by increasing their awareness of impairments and of the potential effects of impairments on task completion C. Complements orientation training and environmental control by incorporating procedures to increase adaptive behavior or decrease maladaptive behavior D. Teaches patients to compensate for residual impairments to restructure daily life environment to minimize effects of the impairment

A

What is the final stage of rehabilitation for many patients with traumatic brain injury? A. Community integration B. Orientation training C. Cognitive-communicative interventions D. Memory improvement programs

A

What is the main goal in evaluating a client with a cleft palate? A. Distinguish speech errors that are obligatory and due to structural deviations from learned errors B. Deliver a speech diagnosis C. Determine a surgical need D. Compare to development norms

A

When establishing eye contact and eye gaze between the caregiver and infant, which of the following is not a type of gaze? A. Interactive gaze B. Social smile C. Gaze coupling D. Diectic gaze

A

When giving treatment in extraclinical settings, such as the playground or classroom, which of the following is best to avoid? A. Bringing attention to the treatment B. Involving spontaneous and functional communication C. Training other professionals and family members D. Using informal training

A

When screening in a school setting, what is one of the most important steps in the identification process of a language disorder? A. Obtaining information from the teacher on how the language impairment affects academic performance B. Seeing how the child interacts with peers C. Identifying the specific language disorder D. Observing pragmatic behaviors during the screening

A

When sound waves move from one medium (e.g., air) to another (e.g., water), the motion causes a bending of the sound wave due to change in its speed of propagation. This phenomenon is known as A. refraction. B. reflection. C. compression. D. rarefaction.

A

Which articulation therapy approach emphasizes both the syllable as the basic unit of speech and the concept of phonetic environment? A. McDonald's sensory-motor approach B. Irwin and Weston's paired stimuli approach C. Baker and Ryan's Monterey Articulation Program D. Van Riper's traditional approach

A

Which of the following could be a technique used in therapy when treating a pediatric case of tongue protrusion/thrusting? A. Change feeing positions; try head slightly reclined. B. Encourage lip closure around the spoon. C. Firmly hold the jaw in place. D. Apply tapping of the lips and nose.

A

Which of the following is a primary characteristic of hyperkinetic dysarthria? A. Involuntary movements B. Weakness C. Reduced range of movement D. Difficulty with motor programming

A

Which of the following is a prosthetic effort that is appropriate for patients with ataxic dysarthria? A. Neck brace or cervical collar B. Nose clip or nasal obturator C. Palatal lift D. Vocal amplifier

A

Which of the following is a theory about the lack of responses from younger children in speech sound development? A. The lack of responses may indicate that younger children avoid sounds that are not in their inventory. B. There is no legitimate reason for the lack of responses from younger children. C. The lack of responses may indicate that younger children are not cognitively developed enough to comprehend certain sounds. D. The lack of responses may indicate that younger children do not have the hearing capabilities to have an extensive inventory.

A

Which of the following is an approach that promotes natural, functional, and conversational communication for teaching language to children? This approach uses natural consequences as reinforcers. A. Naturalistic child language teaching method B. Milieu teaching C. Mand model method D. Direct language treatment approaches

A

Which of the following is considered a technique for the treatment of articulation and phonological disorders for a patient with cerebral palsy A. Modifying or eliminating inappropriate and ineffective compensatory postures B. Assessing the child's developmental milestones C. Treating velopharyngeal incompetence D. Considering how much the brain damage has affected motor planning

A

Which of the following is considered a tongue thrust? A. A deviant swallow in which the tongue is pushed forward against the central incisors B. Simultaneous use of multiple modes of expression to enhance communication C. Normal tongue movement associated with swallowing D. Rapid, small movements of the tongue during purposeful activity

A

Which of the following is not a common etiology of hyperkinetic dysarthria? A. Parkinson's disease B. Huntington's disease C. Seizure disorders D. Brainstem stroke

A

Which of the following is not true about theW fringe vocabulary set of an augmentative and alternative communication device? A. it includes words that occur frequently in typical speech B. it typically includes nouns C. it includes vocabulary specific to the individual D. it may include names of people or places

A

Which of the following procedures involves a placement of a small flexible prosthesis into a surgically placed puncture in the patient's stoma? A. Tracheoesophageal puncture B. Esophageal puncture C. Electrolarynx procedure D. Staffieri neoglottis procedure

A

Which type of reinforcer does not rely on past learning and fulfills biological needs? An example is food or drink. A. Primary reinforcer B. Secondary reinforcer C. Conditioned reinforcer D. Social reinforcer

A

While other factors may be involved, what aspect of a client with cleft palate will have the most obvious effect on the developing sound system prior to palatal surgery? A. Clefting B. Hearing loss C. Language disorders D. Voice disorders

A

With different types of dysarthria, a differential diagnosis may be challenging. The clinician needs to know the unique features of each type. Obviously, each type may be contrasted on more than one feature, but which of the following statements correctly contrasts two types to help make a differential diagnosis? A. Excessive and even stress helps distinguish ataxic dysarthria from hyperkinetic dysarthria, with its equal stress B. Imprecise production of consonants help distinguish hyperkinetic dysarthria from hypokinetic dysarthria, which is characterized by distorted vowels. C. Impression of drunken speech found in mixed dysarthria helps distinguish it from ataxic dysarthria, in which that characteristic is absent. D. Excessive loudness variations of flaccid dysarthria distinguish it form monoloudness of hyperkinetic dysarthria.

A

With different types of dysarthria, a differential diagnosis may be challenging. The clinician needs to know the unique features of each type. Obviously, each type may be contrasted on more than one feature, but which of the following statements correctly contrasts two types to help make a differential diagnosis? A. Excessive and even stress helps distinguish ataxic dysarthria from hyperkinetic dysarthria, with its equal stress. B. Imprecise production of consonants helps distinguish hyperkinetic dysarthria from hypokinetic dysarthria, which is characterized by distorted vowels. C. Impression of drunken speech found in mixed dysarthria helps distinguish it from ataxic dysarthria, in which that characteristic is absent. D. Excess loudness variations of flaccid dysarthria distinguish it from monoloudness of hyperkinetic dysarthria.

A

Within Huntington's disease, there are two neurotransmitters responsible for controlling choreiform movements. Choreiform movements mimic dance. What are the two neurotransmitters lacking in a person with Huntington's disease? A. Acetylcholine and gamma amino butyric acid (GABA) B. Acetylcholine and dopamine C. Dopamine and gamma amino butyric acid (GABA) D. Gamma amino butyric acid (GABA) and serotonin

A

You are evaluating Tatyana, a Russian-speaking child with a suspected language impairment. To accurately estimate her language skills, you engage a Russian-speaking interpreter and count Tatyana's responses to test questions in both Russian and English. You are employing: A. conceptual scoring B. dynamic assessment C. criterion-referenced testing D. assessment of working memory

A

You are evaluating a basketball coach, Susan, who tells you, "I am having problems with my voice." You notice intermittent, involuntary, fleeting vocal fold abduction when she tries to phonate. This is known as A. abductor spasmodic dysphonia. B. adductor spasmodic dysphonia. C. bilateral paralysis. D. unilateral paralysis

A

You are evaluating the language skills of Pascal, who has come to the United States withhis family from Mexico. As you are gathering a language sample, you hear many utterancesthat reflect the transfer of Spanish to English. Which of the following would be a typicalutterance for a child from a Spanish-speaking home who is learning English in elementaryschool? A. "This balloon is more big." B. "I don't not have no more balloons." C. "The big house be red." D. "The girl's book done be gone."

A

Your assessment of a child with cerebral palsy (CP) shows that in addition to significant speech and language problems, the child exhibits slow, writhing involuntary movements. You correctly diagnose that the child has which of the following? A. Athetoid CP B. Ataxic CP C. Quadriplegia D. Spastic CP

A

Joint attention is very important when establishing the interaction between the caregiver and infant. Caregivers and infants have constant joint attention. What are the two types of joint attention that occur between caregivers and infants. A. joint reference B. joint action C. joint preference D. joint perception

A, B

To understand a spectrogram during analysis of a sound spectrum, there is an x axis and a y axis. What two measurements are needed for either the x axis or y axis? A. Time B. Amplitude C. Frequency D. Intensity

A, C

What are the two most prominent air-filled cavities that compose the outer ear and have a resonant frequency to which they respond best? A. Concha B. Tympanic cavity C. Ear canal D. Ossicles E. Helix F. Anti-helix

A, C

When treating a client, it is important to establish baselines for target behaviors. Which two of the following choices describes a specific procedure that establishes baselines of target behaviors in which multiple attempts to produce a target response are counted separately and trials are separated in time? A. Baselines B. Evoked trials C. Discrete trials D. Modeled trials E. Conversational speech

A, C

You evaluated a child and have made the following conclusions: [r] is not stimulable at any level, [s] and [z] are stimulable at the sound and word level, and [θ] and [ð] are stimulable at the sound level. What two sounds should be initiated first in trial probe therapy? A. [s] B. [r] C. [z] D. [θ] E. [ð]

A, C

A treatment procedure is selected or rejected based on the level of evidence of the particular technique. What level of evidence is described if a technique has been experimentally evaluated with a control group or single-subject design with control conditions? A. Unreplicated B. Replicated C. Uncontrolled case studies D. Controlled experimental studies

A, D

A 2-year-old client with delayed language was added to your caseload. Following initial introductions with the client's mother, she noted that he recently started having difficulties with feeding and breathing. You complete an oral mechanism examination and notice that his tongue is positioned posteriorly (glossoptosis), covering his airway. He has an unusually small mandible and has incomplete closure of the roof of the mouth (cleft palate). He also presents with a unilateral conductive hearing loss. This patient should receive further evaluation, but the symptoms indicate that he has: A.Landau-Kleffner syndrome B.Pierre-Robin syndrome C.Fragile X syndrome D.Moebius syndrome

B

A 49-year-old patient was involved in a motor vehicle accident and suffered damage to the cerebellum or brainstem vestibular nuclei. He was experiencing problems with articulation and prosody. He also exhibited slurred speech and discoordination of sounds. His direction, force, and timing of movements were affected. The physician diagnosed him with: A. Hyperkinetic dysarthria B. Ataxic dysarthria C. Flaccid dysarthria D. Spastic dysarthria

B

A 70-year-old male has suffered a left cerebrovascular accident (CVA). His verbal output is considered to be fluent; however, he exhibits phonemic and semantic paraphasias, neologisms, empty speech, and lacks content words. He most likely presents with: A. Broca's aphasia B. Wernicke's aphasia C. Anomic aphasia D. Apraxia

B

A 73-year-old Cantonese-speaking gentleman, Mr. Fung, has had a stroke. You are seeing him for therapy in an outpatient rehabilitation setting. He is recovering both his Cantonese and his English skills, but you are conduciting therapy in Encligh only because a Cantonese-speaking speech-language pathologist is not available. Which one of the following productions would be an example, on Mr. Fung's part, of English influenced by his primary language of Cantonese ad not necessarily his current neurological status? A. "He not have no most money in his pocket." B. "She coming over here now." C. "We don't no done got to have breakfast now." D. I done axed them for help."

B

A child from Culber City transfers to Central City, and his file indicates that he has been receiving speech-language services in Culber City. Some pages of the report from the speech-language pathologist in Culber City are missing. However, the first page indicates that this child has Moebius syndrome. He also has a history of frequent hospitalizations. What can the Central City clinician probably expect to find? A. This child has syndactyly, cranial synostosis, resulting in smaller anterior-posterior skull diameter, high forehead, an arched and grooved hard palate, class III malocclusion, and possible cleft of the hard palate. B. This child has delayed language and an articulation disorder, as well as bilabial paresis and weak tongue control for lateralization, elevation, depression, and protrusion, a mask-like face, a history of feeding problems in infancy, and unilateral or bilateral paralysis of the abductors of the eye. C. This child has a small maxillary structure, sphenoethmoidal synchondroses, ocular hypertelorism, facial asymmetry including a tall forehead, and brachycephaly. D. This child has low muscle tone, a history of early feeding difficulties, initial failure to thrive, obesity after the first year, and underdeveloped genitals.

B

A child who regularly says, "He the small one," instead of "He is the smallest one," or "She is tall than her" instead of "She is taller than her" has specific problems with A. adjectives B. comparatives and superlatives C. irregular past-tense forms D. word retrieval

B

A child with cerebral palsy is seen for an evaluation. During the assessment, Cara, the speech-language pathologist notices that the child's jaw is impaired relative to other structures. Cara is trying to focus on improving the child's speech by increasing tongue movement to improve articulatory proficiency. Cara would use the following to determine whether the child would be a candidate for constraint-induced movement therapy: A. Modified barium swallow study B. Bite block C. Tongue depressor D. Endoscopic evaluation

B

A client comes to your office for an evaluation and provides you with reports from other professionals who have previously seen the client. How you should use this information? A. The information most likely creates a biased view of a client's condition and should not be heavily considered. B. You should consider the information but rely primarily on your own direct observation and evaluation results. C. If the information identifies specific treatment options and alternatives, you should use these options and alternatives for the client. D. The information is not legal to obtain because it is from another professional, which means you should not use it to help you evaluate your client.

B

A father comes to you regarding his daughter, who is 8 months old. The daughter's hearing loss is bilateral, and she is profoundly deaf. The father states that he wishes for his daughter, as she grows older to "fit in with children with normal hearing." He is interested in any possible amplification and says that he wants his daughter to lead a life that is "as normal as possible." Which training approach would best fit this father's wishes? A. Total communication B. Aural/oral method C. Manual approach D. Rochester method

B

A father comes to you regarding his daughter, who is 8 months old. The daughter's hearing loss is bilateral, and she is profoundly deaf. The father states that he wishes for his daughter, as she grows older, to "fit in with children with normal hearing." He is interested in any possible amplification and says that he wants his daughter to lead a life that is "as normal as possible." Which training approach would best fit this father's wishes? A. Total communication B. Aural/oral method C. Manual approach D. Rochester method

B

A friend asks you about her baby Sasha, who is being exposed to Croatian and English in the home. Your friend explains that Sasha has heard both Croatian and English from early infancy. Sasha is experiencing the phenomenon of: A. Sequential bilingual acquisition B. Simultaneous bilingual acquisition C. Limited bilingualism D. Dual language immersion

B

A high school teacher has referred Ashley to you for assessment. Ashley is 17 years old, and she possibly has a language impairment. When you assess Ashley, which specific problems will you look for as evidence of a language impairment? A. Sentences that are average in length but contain omissions of bound morphemes B. Difficulty using cohesion devices (e.g., therefore, for example) C. Normal ability to maintain a conversation, but some use of non sequiturs D. Normal ability to use figurative language, but difficulty with concrete language

B

A new client comes to your office with severe allergies as one of the medical conditions listed on his case history form. Which of the following statements best describes why this piece of information would be important to you as a speech-language pathologist? A. Allergies are not important because of their high prevalence but they should still be listed on his case history form for other professionals such as physicians. B. They can have potential implications for the development of a speech and language disorder. C. Allergies cause a fluid buildup in the middle ear, which results in a hearing loss and speech and language difficulties. D. Allergies result in airway inflammation, which hurts the sound quality of the voice.

B

A patient complains that her voice is soft, hoarse, low pitched, and breathy. After conducting an endoscopic evaluation, the laryngologist concludes that the patient has benign growths of thick, whitish patches on the surface membrane of the mucosa. The laryngologist asks his resident to diagnose the problem. The resident identifies the problem as A. hyperkeratosis B. leukoplakia C. hemangioma D. a granuloma

B

A patient was having difficulty swallowing solids and liquids and was regurgitating his food hours after eating. During a barium swallow the physician noted that there was a complete loss of peristalsis. She also noted that the nonrelaxing lower esophageal sphincter (LES) was preventing the downward passage of the bolus into the stomach. This condition is called A. trismus B. achalasia C. luminal deformity D. extrinsic compression

B

A professor is teaching a class and informs his students that ________ interference reduces the amplitude of the resultant wave. A. constructive B. destructive C. optical D. quantum

B

A screening test for dementia is: A. Arizona Battery for Communication Disorders of Dementia (ABCD) B. Short Portable Mental Status Questionnaire (SPMQ) C. Functional Linguistic Communication Inventory (FLCI) D. Dementia Mood Assessment Scale

B

A speech-language pathologist on a cleft palate and craniofacial team wishes to develop a simple measure of hypernasality to begin quantifying (however subjectively) the amount of hypernasality he hears in the speech of the children seen by the team each month. He will pass on this information to the plastic surgeon and other team members to assist them in making surgical decisions for each child. The speech-language pathologist devises the following scale 1 almost no hypernasality 2 slight hypernasality 3 moderate hypernasality 4 great amount of hypernasality What is this type of scale called? A. Logarithmic scale B. Ordinal scale C. Nominal scale D. Interval scale

B

A speech-language pathologist should consider which of the following to include as part of general principles regarding treatment of cerebral palsy? A. Cerebral palsy is a unique speech disorder and generalized treatment approaches that are used for other children can be applied to treat these children. B. Make a thorough assessment of communication problems and design treatments to suit the child's problems, needs, and strengths. C. Disregard educational demands made or to be made on the child. D. Prescribe pharmacological intervention.

B

A speech-language pathology graduate student received her first adult patient diagnosed with apraxia of speech. The student consulted with the supervisor, who recommended that they try to maintain the natural prosody of the patient's utterances. She suggested using computer-generated pacing tones each time the patient produced an utterance, so that the patient could keep the natural rhythm of the utterance without regard to the rate of speech. The student clinician conducted further research and during therapy coupled hand-tapping and choral reading along with the suggestions made by the supervisor. This type of therapy is called A. metronomic pacing. B. metrical pacing. C. cued speech. D. integral practice.

B

All of the following are communication disorders associated with cleft palate, except: A. Hearing loss B. Fluency disorders C. Speech sound disorders D. Language disorders

B

All of the following could be techniques used in therapy when treating a pediatric case of tongue retracting except? A. Alter feeding position, try head slightly flexed with close chin-down position. B. Begin at the middle of the upper lip and tap to the corner of the mouth; repeat on the other side. C. Alter feeding utensil; try a longer nipple. D. Apply rhythmic tapping under base of chin.

B

Among the following symptoms, which one is not an especially significant feature of traumatic brain injury in children? A. sentence comprehension problems B. marked deficiency in producing grammatical morphemes C. topic maintenance D. word retrieval problems

B

Among the following symptoms, which one is not an significantly significant feature of traumatic brain injury in children? A. Sentence comprehension problems B. Marked deficiency in producing grammatical morphemes C. Topic maintenance D. Word-retrieval problems

B

An ABAB design is a way to evaluate treatment effects through a single-subject research study in which the target behavior is: A. base rated, taught with the technique to be evaluated, then reduced by taking away treatment to confirm that teaching was efficient. B. base rated, taught by employing a treatment program, then the treatment is withdrawn, and finally the treatment is reapplied to confirm that teaching was effective. C. base rated, reduced by taking treatment away, then taught again to confirm that teaching was efficient. D. base rated, taught by employing treatment, reduced through the instruction of its counterpart, then extinguished.

B

An audiologist is testing a client for acoustic immittance. She places a sound stimulus in her client's external ear canal with an airtight closure and measures changes in the acoustic energy as the sound stimulates the auditory system. What instrument is the audiologist using for this particular procedure? A. Otoscope B. Impedance bridge C. Audiometer D. Tympanic meter

B

An instrument used to determine velopharyngeal function is called what A. Accelerometer B. Nasometer C. Velar closure meter D. Manometer

B

An opera singer complains that she is unable to maintain adequate breath support to produce her optimal voice. You want to measure the singer's lung volume to check for adequate breath support for optimal voice. You will need to measure ___________, which is the volume of air that the singer can exhale after a maximal inhalation. A. tidal volume B. vital capacity C. total lung capacity D. residual air

B

Annette, a 56-year-old woman, suffers from a lesion covering a large portion of her Perisylvian area, caused by total occlusion of the left middle cerebral artery. She displays deficits in all aspects of language. Her speech is nonfluent and she experiences severe sensory and motor deficits on the complete right side of her body. She has limited use of her automatic language. She most likely presents with: A. Anomic aphasia B. Global aphasia C. Mixed transcortical aphasia D. Broca's aphasia

B

As a clinician in a medically based private practice, you receive a referral of 23-year-old Allison, a college cheerleader. Allison has been a cheerleader since her freshman year at Freeport College; she is now a senior. She works part-time as a telemarketer, and, according to her boyfriend, she "is glued to her cell phone." She also sings in the college chorus. She has been hoarse for several years and tells you during the case history, "I've ignored the way I sound—it's just me. I haven't felt like I've needed to change anything." However, she shares that lately she has been feeling a lot of pain and the hoarseness is substantially worse. She says, "Sometimes when I talk, it's almost like there's a 'double voice.'" She tells you that she is worried because she will graduate from college in 3 months and will be looking for a job. She is worried that employers will not want to hire someone who "sounds like a frog." You immediately refer her to an otolaryngologist for a thorough examination of her vocal folds. You then proceed to do your own instrumental and perceptual evaluation. You come up with a number of findings, including the fact that Allison has increased laryngeal airway resistance, a maximum phonation time of 6 seconds, and dysphonia. You think that she is a probable candidate for phonosurgery but will wait for the otolaryngologist's diagnosis and recommendations. Measures of jitter and shimmer are becoming more common in use with voice patients because they can be useful in early detection of vocal pathology. Although you suspect that the otolaryngologist will find obvious vocal pathology, given Allison's history of prolonged hoarseness, you still want to obtain measures of jitter and shimmer because these can serve as an excellent baseline, especially if Allison has phonosurgery. When you take these measures, you might expect to see A. a small amount of shimmer and a large amount of jitter. B. large amounts of both jitter and shimmer, with more than 1 dB of variation across vibratory cycles when shimmer is measured. C. large amounts of both jitter and shimmer, with more than 1 dB of variation across vibratory cycles when jitter is measured. D. large amounts of both jitter and shimmer, with Allison being able to sustain a vowel with approximately 15% shimmer.

B

Attachment is a close, nurturing, long-term relationship that develops between the caregiver and infant. Interaction between caregiver and infant strengthens this attachment. When speaking to the caregiver before therapy, it is important to establish the infant vocalizations and Motherese between the caregiver and the infant. All of the following are examples of basic cries except: A. Hunger B. Happy C. Pain D. Anger

B

Barry, a 42-year-old male, slips on ice and hits his head on a concrete sidewalk. He does not lose consciousness but has some swelling and a small laceration at the back of his head. He appears to be fine and has been moving around. Later, he complains of a severe headache and takes some aspirin. Forty-five minutes after taking the aspirin, he still has a headache and is feeling nauseous. Shortly after this he vomits and is complaining of a stiff neck. What neurologic event might explain Barry's symptoms? A. Epidural hemorrhage B. Subdural hemorrhage C. Subarachnoid hemorrhage D. Intracerebral hemorrhage

B

Debbie, an 80-year-old female who recently underwent brain surgery, is presenting with imprecise and weak pressure consonants, diminished reflexes, and hypotonia. Based on these symptoms, your patient is most likely presenting with which of the following types of dysarthria? A. Ataxic dysarthria B. Flaccid dysarthria C. Hyperkinetic dysarthria D. Hypokinetic dysarthria

B

Evaluating spelling proficiency can actually provide valuable diagnostic information. Poor spelling may reveal weaknesses in the following linguistic components except: A. Orthographic knowledge B. Pragmatic knowledge C. Morphologic knowledge D. Phonemic awareness

B

Human communication is vital for language development. Which of the following is not a main cognitive principle? A. Equilibrium B. Imitation C. Organization D. Adaptation

B

If a client is having difficulty producing a word with a vowel, it is important for the clinician to distinguish whether the vowel is tense or lax. What is the best way for a clinician to quickly discern a lax vowel from a tense vowel? A. Look at whether the vowel is rounded or unrounded B. Tense vowels are relatively long in duration C. Lax vowels are relatively long in duration D. Tense vowels are relatively short in duration

B

If a screening test for a specific disorder has high sensitivity and low specificity, what does this reveal about the test? A. It may omit the condition in favor of individuals who are free of the disorder. B. It may overidentify the cases of the disorder. C. It may not identify individuals who truly have the disorder. D. The test correctly measures the chances that an individual has the disorder

B

If a screening test for a specific disorder has high sensitivity and low specificity, what does this reveal about the test? A. It may omit the condition in favor of individuals who are free of the disorder. B. It may overidentify the cases of the disorder. C. It may not identify individuals who truly have the disorder. D. The test correctly measures the chances that an individual has the disorder.

B

In assessing a patient with swallowing disorders, you would A. not be concerned about screening the patient for concrete and abstract language comprehension, because it would provide irrelevant information B. consider the correct positioning of the patient for certain procedures C. restrict the patient's food choices to limit potential aspiration D. not consider literacy skills, because they are irrelevant to swallowing assessment

B

In the mans-model method of teaching language skills to children, the clinician does which of the following? A. Prompts an elaboration of a verbal response, and requires the child to imitate. B. Asks questions like "Tell me what you want" and models or prompts the correct response if necessary C. Adds comments to the child's production; with no response requirement D. Repeatedly and consecutively models a particular structure but does not require the child to imitate it

B

In the source-filter theory, the source and the filter are assumed to be independent. It is implied that you can change the output of the source without changing the filter and vice versa. Which anatomical structure represents the source? A. Lungs B. Vocal fold vibration C. Vocal tract D. Vocal folds

B

In this approach to counseling, clients need acceptance and positive unconditional regard to develop congruence between their self-concept and behavior: A. Psychodynamic theory B. Client-centered theory C. Behavioral theory D. Cognitive-behavioral theory

B

Janine, an 83-year-old patient, is attending therapy for memory deficits caused by Alzheimer's disease. Janine's current goal is to use her techniques to recall a three-point plot in a short video. Throughout the video clips, she talks about the plot. What direct intervention is Janine utilizing? A. Multimodal approach B. Self-generated cues C. Spaced retrieval training D. Direct priming

B

Joe, a 6-year-old boy, was being taught to use an augmentative and alternative communication (AAC) device. On his communication device, a hieroglyphic picture of a cat was displayed. This would be considered an: A. Arbitrary symbol B. Iconic symbol C. Abstract symbol D. Anagrammed symbol

B

Kinetic energy is energy that is being used. Over time, kinetic energy is transformed to thermal energy and the result of this is damping or damped vibration. What is damping? A. Periodically allowing motion to pass B. Opposition to motion C. Opposition to frequency D. Periodically controlling energy

B

Macy, 2-year-old girl, was referred to you by her mother because she was unintelligible. During Macy's oral examination, you discover a labioversion of her incisors. Macy's masseter muscle is evidently weak as she swallows and she has a disorganized method for creating a bolus. Macy also has an open bite. After consideration of all visible symptoms, you determine that this Macy has: A. Ankyloglossia B. A tongue thrust C. Class I Malocclusion D. Class II Malocclusion

B

One criterion for a patient to receive a supraglottic laryngectomy is that they must have the potential to: A. use an electrolarynx. B. relearn a swallowing sequence. C. occlude a stoma to be able to speak. D. swallow without aspiration immediately following surgery

B

One year post cleft lip and palate surgery you are revaluating a 3-year-old patient. It appears that after her surgery, she does not have any medical difficulties; however, she has mild hypernasality. Her family notes that they would be more comfortable addressing her mild hypernasality during speech therapy rather than getting surgery. Which of the following best describes the information that you should share with your patient's family? A. Surgical procedures to correct structural abnormalities is the only way to correct her mild hypernasality. B. Voice therapy could be provided to address her mild hypernasality. C. Articulation therapy could be provided to address her mild hypernasality. D. There is no treatment to reduce hypernasality.

B

Pablo is a Spanish-speaking third grader who is in the process of learning English. His parents came to the United States 1 year ago from Mexico. His classroom teacher refers him for a speech evaluation, saying that he "sounds different—I think he may need speech therapy." When you evaluate Pablo's speech, you hear the following patterns. Which one of them would not be typical for a Spanish-speaking student in terms of predictable productions based on Spanish influence? A. t/th substitutions (e.g., ting/thing) B. ch/f substitutions (e.g., chan/fan) C. a/ae substitutions (e.g., block/black) D. Devoicing of final consonants (e.g., luff/love)

B

Patients who have damage to their nerve fibers along the ascending auditory pathways from the internal auditory meatus to the cortex have a A. cochlear disorder B. retrocochlear disorder C. recruitment disorder D. middle ear disorder

B

Paul is a 60-year-old educator who has recently been experiencing symptoms of disequilibrium. For example, his wife explains that he demonstrates unsteady walking or abrupt and unexplained falls without loss of consciousness. Paul also complains about his vision—indicating that it is blurry and he is sensitive to light. Most recently, he has been experiencing slurred speech and various mental complaints such as slowness of thought, impaired memory, personality changes, and changes in mood. Based on Paul's symptoms, what is his diagnosis likely to be? A. Essential tremor B. Progressive supranuclear palsy C. Vascular parkinsonism D. Multiple system atrophy

B

Peter and Melissa, who are family friends, come to you for advice regarding the development of their 6-month-old child. The parents mentioned that their daughter Jaqueline was born prematurely. The parents are concerned because all their other children were already babbling with intonation prior to 6 months of age. Peter mentioned that she was babbling, but her voice was "very flat when she talked to Mommy and Daddy." What would you recommend to Peter and Melissa? A. That the jargon stage is when children begin to develop prosodic elements in their speech. The average age range during which children reach and complete this stage is about when they are 10 to 12 months old. They should not worry about their child's development until she turns 1; she still has a few months to master jargon. B. That the canonical babbling stage is when children start to use prosodic elements in their babbling. The average age range during which children reach and complete this stage is about when they are 6 to 10 months old. They should not be worrying about their child's development until she surpasses 10 months; she still has a few months to master canonical babbling. C. That the canonical babbling stage is when children start to use prosodic elements in their babbling. This stage begins at 6 months for all healthy children; therefore, since the child is 6 months old and has not reached this stage yet and was born prematurely, the parents should seek further help. D. That the jargon stage is when children start to use prosodic elements in their babbling. This stage begins at 6 months for all healthy children; therefore, since the child is 6 months old and has not reached this stage yet and was born prematurely, the parents should seek further help.

B

Rongomaiwhenua was diagnosed with apraxia of speech (AOS). She displays an inability to follow commands and perform voluntary speech movements. Recently, she has been diagnosed with nonverbal oral apraxia (NVOA), a diagnosis that often co-occurs with AOS. With this diagnosis, it can be expected that a lesion would most likely exist in the: A. Posterior portion of the insula B. Frontal and central opercula C. Third temporal convolution D. Piriform cortex

B

Select the correct statement. A. Probes and baselines are the same. B. Baselines are pretreatment measures, and probes measure generalized productions. C. Probes are conducted only in naturalistic settings (e.g., homes). D. Baselines are measures of generalized productions.

B

Select the statement that is true of conduction aphasia. A. It is caused by widespread lesions in the entire perisylvian region. B. It is characterized by good syntax, prosody, and articulation. C. Its lesion sites are more definitive than for other types of aphasia. D. Speech associated with it is as fluent as it is in Wernicke's aphasia

B

Select the statement that is true of conduction aphasia. A. It is caused by widespread lesions in the entire perisylvian region. B. It is characterized by good syntax, prosody, and articulation. C. Its lesion sites are more definitive than for other types of aphasia. D. Speech associated with it is as fluent as it is in Wernicke's aphasia.

B

Select the statement that is true. A. To encourage clients to seek treatment, prognosis should always be stated in optimistic terms. B. Prognosis should be stated based on assessment data and available research and clinical data. C. Prognosis always refers to what happens if treatment is not offered. D. It is not necessary to offer prognostic statements to all clients.

B

Siobhan, a 74-year-old female, is a new resident at the nursing home. After reading her case history, you learn that she recently has had a history of multiple strokes. In her case history, it states that her cognitive impairments coincided with her strokes. She is disoriented, confused, has trouble speaking or understanding speech, and has vision loss. Other symptoms include uncontrolled laughing and crying, problems with judgment and planning, and a reduced ability to pay attention. Her daughter reports that Siobhan appears to display symptoms of depression, noting that "ever since the strokes, Mom has a difficult time with simply getting up in the morning and she just doesn't seem to enjoy being around her family like before." In person, Siobhan shows loss of motivation and attention during various tasks. She appears to have some memory loss, but primarily struggles with executive functioning. Siobhan also presents with small-step gait and unsteadiness. She is prone to falls and is required to walk with a cane. Her regular nurses report that she has urinary urgency not explained by urologic disease along with emotional incontinence. Due to concerns expressed by her family, Siobhan is being evaluated to determine the presence of dementia. Based on the information provided, Siobhan's most likely presents with: A. Traumatic dementia B. Vascular dementia C. Toxic dementia D. Lewy body dementia

B

Some total laryngectomy patients present with a fold of tissue at the base of the tongue, which can have an impact on swallowing following surgical intervention. The fold of tissue is either mucous membrane or scar tissue that originates from the lateral pharyngeal wall and travels into the base of the tongue. This condition is called: A. prosthesis B. pseudoepliglottis C. aryepiglottic fold D. false vocal folds

B

Speech samples of persons who clutter may include deletion of non-stressed syllables in longer words (e.g., defly for definitely) and is an example of A. spoonerism. B. cluttering. C. stuttering. D. tachylalia.

B

The cerebral palsies originate in infancy, and the speech impairment that may coexist with the movement disorder is also considered ________________. A. Verbal apraxia B. Dysarthria C. Dysphagia D. Neuromuscular difficulties

B

The components of an AAC system are crucial to consider during an assessment. When assessing a young child, you evaluate the way the individual will use and interact with the system, including the user interface, selection method, and output. What type of components are being described? A. Primary components B. Secondary components C. Tertiary components D. Global components

B

The cuneiform cartilages are embedded within the: A. Thyromuscularis B. Aryepiglottic folds C. Cricoid cartilage D. Thyrovocalis

B

The degree to which an instrument measures what it purports to measure is known as A. reliability. B. validity. C. concurrency. D. measurability

B

The degree to which an instrument measures what it purports to measure is known as A. reliability. B. validity. C. concurrency. D. measurability.

B

The following characteristics are important regarding the adaptation effect in persons who stutter, except: A. The greater the time interval between readings, the less the degree of adaptation. B. All persons who stutter exhibit an adaptation effect. C. Adaptation is generally seen in both persons who stutter and typically fluent speakers. D. There is no transfer of adaptation from one passage to another.

B

The radiographic imaging procedure that allows X-ray beams to circle through segments of the brain and pass through tissue while a camera takes pictures of sections of the structures being scanned is known as A. electroencephalography (EEG). B. computerized axial tomography (CAT) scan. C. magnetic resonance imaging (MRI). D. positron emission tomography (PET) scan.

B

The tensor tympani muscle in the middle ear is innervated by cranial nerve: A. I B. V C. VIII D. IX

B

The tensor tympani muscle in the middle ear is innervated by cranial nerve: A. I B. V C. VIII D. IX

B

There are four guidelines for designing a care plan that maximizes overall patient function. Which of the following guidelines would not be helpful for patient function during a treatment session? A.Strengthen knowledge to potentially improve function. B. Increase demands on impaired cognitive systems. C. Provide stimuli that evoke positive memory and emotion. D. Promote the use of intact cognitive systems

B

There are two broad levels of human communication: verbal communication and nonverbal communication. Verbal communication uses words as symbols to exchange ideas. Nonverbal communication includes many behaviors that communicate with or without the associated production of symbols. Which of the following is not an example of nonverbal communication? A. Facial expression B. American Sign Language C. Body language D. Extralinguistic communication

B

There is limited epidemiological research with regard to risk factors for developing cluttering; however, several experts have indicated that potential risk factors may include the sex of the child with cluttering more common in males than females and the male to female ratio ranging from: A. 2:1 to 5:1 B. 3:1 to 6:1 C. 4:1 to 7:1 D. 5:1 to 8:1

B

Tonya is a 4-year-old who presents with muscle stiffness in her legs. Her arms are not affected at all. Tonya struggles to walk because her tight hip and leg muscles cause her legs to pull together. What type of spastic cerebral palsy does Tonya have? A. Spastic hemiplegia B. Spastic diplegia C. Spastic quadriplegia D. Spastic triplegia

B

Vijay, a 70-year-old man, attends a clinic for a hearing evaluation. After conducting testing, the audiologist mentions that a common cause of conductive hearing loss where there is a buildup of spongifying bone on the osseous labyrinth that immobilizes the footplate of the stapes and interferes with sound transmission to the inner ear is: A. Meniere's disease B. Otosclerosis C. Otitis media D. Acoustic neuroma

B

What are the developmental domains that IDEA Part C requires regarding early intervention evaluation? A. Cognitive, communication, and physical development B. Social-emotional, communication, physical, adaptive skills, and cognitive development C. Physical, communication, cognitive, and social-emotional development D. Communication, physical, adaptive skills, and cognitive development

B

What is a major cause of death and disability in Americans younger than 35? A. Parkinson's disease B. TBI C. Huntington's disease D. Stroke

B

What is the procedure that uses a pulsing light to permit the optical illusion of slow-motion viewing of the vocal folds? A. Electroglottography B. Stroboscopy C. Electromyography D. Videofluoroscopy

B

What type of partial laryngectomy procedure is used when there is a smaller lesion on the supraglottic larynx? A. Vertical or subglottic laryngectomy B. Horizontal or supraglottic laryngectomy C. Lateral or supraglottic laryngectomy D. Medial or glottic laryngectomy

B

When analyzing spectrograms on a spectrograph, there are different types of spectrograms that allow you to analyze your sample to the best of its ability. These different types of spectrograms are called wideband spectrograms and narrowband spectrograms. Which of the following characteristics is expressed better in a wideband spectrogram? A. Long time window B. Short time window C. Good for showing harmonics D. Good for measuring harmonics

B

When assessing the communicative disorders of your 10-year-old client Sofia, you use charting—which provides a method of scoring her responses and identifying herabilities/deficits. Choose the incorrect statement about charting: A. If Sofia stays in her chair for 30 seconds, this is an example of a behavior that is worth charting. B. Speech pathologists use one way to chart behaviors and that is by noting each time a preselected behavior is exhibited. C. Even if certain behaviors are not necessarily caused by the communicative disorder, charting can still be appropriate for behaviors that are important in the treatment process. D. Different forms are available that are designed for specific age groups.

B

When screening children and assessing their communication, it is important to assess sensorimotor foundations for language. Means-end is an example of a sensorimotor foundation for language. What is meant by the term means-end? A. Representing one thing by using something else B. Attaining a desired goal through purposeful action C. Recognizing the behavior of others and reproducing it D. Understanding that things still exist when not in sight

B

When the wavelength of a sound is longer, the frequency is lower, and the wave ______ more. A. refracts B. diffracts C. fractures D. breaks

B

Which articulation difference is not commonly observed among Asian speakers of English as a second language? A. Shortening of polysyllabic words B. t/k substitution (e.g., tin/kin) C. Confusion of /r/ and /l/ D. Substitution of a/ae (e.g., shock/shack)

B

Which goal of stuttering modification therapy is incorrect? A. reducing the severity of stuttering B. making stuttering less abnormal C. modifying the shape of stuttering D. changing the frequency of stuttering

B

Which legal act dictates procedures for collaboration between various professionals and professional groups to deliver and plan services to students? A. Act 89 B. Individuals with Disabilities Act (IDEA) C. Education for All Handicapped Children Act D. Public Law 94-142

B

Which of the following is a group of genetic skeletal muscle diseases associated with muscle fiber degeneration and replacement with fatty and fibrous connective tissue? A. multiple sclerosis (MS) B. muscular dystrophy (MD) C. Amyotrophic lateral sclerosis (ALS) D. Poliomyelitis (Polio)

B

Which of the following is not a common compensatory gesture seen with clients with cleft palate? A. Pharyngeal fricative B. Pharyngeal glide C. Nasal fricative D. Glottal stop

B

Which of the following is not a likely cause of congenital cerebral palsy? A. Meningitis B. Dementia C. Traumatic brain injury D. Stroke

B

Which of the following is not a treatment focus of speech-language pathologists who work with patients with Parkinson's disease? A. Physical exercise and increased attention to breathing B. Making modifications to feeding utensils because of tremors C. Increase swallowing abilities by altering eating habits D. Making modifications to food consistency

B

Which of the following is not a typical age-related change in the larynx? A. Hardening of the laryngeal cartilages B. Velopharyngeal insufficiency C. Degeneration and atrophy of the intrinsic laryngeal muscles D. Degenerative changes in the lamina propria

B

Which of the following is the most commonly seen speech disorder in individuals with cleft palate? A. A fluency disorder B. A disordered speech sound system C. A voice disorder D. Apraxia

B

Which of the following statements could place limitations on children during the first 50-word stage of development? A. Children have phonetic variability, so they use many unstable productions of phonemes. B. There are a limited number of syllable shapes used. CV, VC, and CVC shapes are mainly used. If any additional syllables are used, there will be unpredictable productions of phonemes. C. It is difficult to take an inventory of a child's production to determine his or her speech sound limitations. D. There is a large amount of variability not only within a child, but across all children in the first-50-word stage. Each child is vastly different, so this causes a problem in finding baseline data.

B

Which of the following would not be a typical predictable production based on Spanish influence? A. Omission of /h/ in word-initial position (e.g., -elp/help) B. w/r substitutions (e.g., wing/ring) C. t/th substitutions in word-initial positions (e.g., tin/thin) D. Devoicing of final consonants (e.g., beece/bees)

B

Which one of the following statements is false? A. An English Learner (EL) student whose background does not match the school's expectations might be inaccurately labeled as having a language impairment. B. If a student has typical learning ability in her primary language but is somewhat slow to learn English, she needs to be placed on the speech-language pathologist's caseload for intervention. C. While learning a second language, children often go through a silent period in which they concentrate on comprehension of the new language. D. A child who said, "Me gustaria manejar; I'll take the car!" is manifesting the process of code switching.

B

With a diagnosis of AOS, it can be expected that a lesion would most likely exist in the: A.Posterior portion of the insula B. Frontal and central opercula C. Third temporal convolution D. Piriform cortex

B

You are administering a formal language test to a student who speaks African American English (AAE). Which one of the following tasks is not biased against him? A. "Tell me if this sentence is correct or incorrect: 'Them kids is havin' fun.'" B. "Tell me what you like to watch on TV." C. "Fill in the missing word: 'Today I walk to school; yesterday I ______ to school.'" D. "Repeat this sentence exactly as I say it: 'They had been careful with their books.'"

B

You are asked to assess a 20-year-old man with traumatic brain injury. Select the following statement that gives you a correct orientation to you assessment of this patient. A. pure linguistic skills, including grammatical skills, need to be assessed in depth B. pragmatic language skills, production of speech sounds, and comprehension of spoken language skills should be among the main targets of assessment C. impaired morphologic skills should be the main concerns of the assessment D. there is no need to spend time on assessing dysarthria

B

You are conducting a fluency evaluation and would like to assess fluency skills and stuttering behaviors in a child who is between 4 and 12 years of age. You would like to identify stuttering, determine the severity of stuttering, as well as document changes in fluency functioning over time. This test examines rapid picture naming, modeled sentences, structured conversation, and narration. The test also includes observational rating scales as well as supplemental clinical assessment. You would select the following test: A. Overall Assessment of the Speaker's Experience of Stuttering (OASES) B. Test of Childhood Stuttering (TOCS) C. KiddyCAT Communication Attitude Test D. Blood's Analysis of Fluency Disorders

B

You are evaluating a 15-year-old patient who was in all-terrain vehicle (ATV) accident and sustained traumatic brain injury (TBI). When evaluating this patient's memory, she mentions that she has no memory of events following the accident. You explain that this may be due to: A. pretraumatic amnesia B. postraumatic amnesia C. orientation to situation D. comprehension deficits

B

You are evaluating a three-year old girl. You would like to assess her speech-associated attitude and are especially interested in the linguistic level of the child. The following assessment would be appropriate to administer: A. Overall Assessment of the Speaker's Experience of Stuttering (OASES) B. Stuttering Severity Instrument 4 (SSI-4) C. KiddyCat Communication Attitude Test D. Speech Locus of Control Scale

B

You are treating a client who was born with a cleft palate. The client had the cleft palate repaired and needs therapy for various articulation and phonological disorders. Which of the following suggestions should be followed when treating children with repaired clefts? A. Start with less visible sounds before the more visible sounds. B. Introduce compensatory articulatory positioning where appropriate. C. Teach glides and liquids before any other sounds. D. Start to train the client using the phonemes /k/ and /g/ if the velopharyngeal functioning is inadequate.

B

You are working in a hospital. A patient, Mr. M., has been referred to you because he is having difficulty adducting his vocal folds. He had surgery for thyroid problems, during which he sustained damage to another structure. Because he has especially been having difficulty with vocal-fold adduction, to what might you suspect that he had damage? A. Ventricular vocal folds B. Recurrent laryngeal nerve C. Superior laryngeal nerve D. External carotid artery

B

You are working in therapy with a child who has challenging behaviors. You decide to use negative reinforcement, which A. reduces the response rate. B. strengthens (increases) responses that terminate or postpone aversive events. C. is comparable to punishment in its effects. D. is involved in differential reinforcement of other behaviors.

B

You evaluate a client who has been in a car accident. The patient complains of significant facial pain, paresthesia (a feeling of pins and needles), and altered sensation of the oral mucosa. You can infer that there has been damage to which cranial nerve? A. Facial Nerve B. Trigeminal Nerve C. Vagus Nerve D. Glossopharyngeal Nerve

B

You have just assessed an 85-year-old man with a high school education, significant hearing loss, poor motor skills, and uncontrolled blood pressure. Your diagnosis is global aphasia. When counseling the family, you inform them that this is the most severe form of aphasia usually caused by a lesion in the left middle cerebral artery. You mention to the family that their loved one has problems with comprehension, speech production, naming, writing, repetition, and reading. At the end of assessment, his wife asks, "What is the prognosis for improvement in communication skills?" What would be an appropriate answer? A. Prognosis is good for significant improvement in communication, provided he receives 3 months of therapy. B. Prognosis is guarded, but I recommend a period of trial therapy because the patient is able to convey information by using picture matching, varying the intonation of his voice, and using simple gestures. C. Prognosis is excellent, as long as you help sustain any improvement gained in therapy. D. Prognosis is unfavorable, so we do not recommend therapy.

B

You plan to write a treatment program for an adult who stutters. You wish to use the fluency-shaping procedure. Among the following choices, what would you include in your program? A. Auditory masking with white noise B. Gentle onset of phonation, slower speech, and normal prosodic features C. Cancellations, pullouts, and preparatory sets D. Fluent stuttering with desensitization to stuttering

B

Your client has a misalignment of the mandibular and maxillary arches and the mandible arch is underdeveloped. Your client has what kind of condition? A. Temporomandibular joint dysfunction B. Congenital mandibular hypoplasia C. Mandibular dislocation D. Class III Malocclusion

B

uring child language intervention, a child says, "Bake cake" and the clinician responds with, "Yes, we are baking a big, pink cake with rainbow sprinkles for your birthday." This is an example of: A. Expansion B. Extension C. Incidental teaching D. Prompting

B

Christopher is an 18-year-old client with a repaired cleft palate. He has production accuracy on most sound classes; however, he still struggles with certain sounds during therapy. Based on his history of a repaired cleft palate, which two of the following sound classes would Christopher most likely have difficulty with? A. Stops B. Fricatives C. Affricates D. Glides

B, C

The following are characteristics of Wilson's disease: A. It is not inherited B. Skin appears yellow C. Kayser-Fleischer rings to appear in the eyes (golden-brown eye discoloration) D. Copper build up in the brain, liver, and vital organs

B, C, D

A child with cerebral palsy typically presents with a host of impairments that can include: A. Dyskinesia B. Intellectual C. Sensory D. Orthopedic E. Myoclonus F. Socioemotional G. Communication

B, C, F, G

In general, which two linguistic and communicative functions appear to be preserved with age? A. Word fluency skills B. Highly overlearned language functions like greetings C. Reading accuracy D. Passive vocabulary/word recognition

B, D

You are consulting with the nursing staff about a patient who is being tube fed. During the evaluation, you notice that the patient is experiencing aspiration. The nurse says that the most appropriate method to reduce aspiration would be to A. introduce a pureed diet. B. elevate the head of the bed at least 30 degrees before and during continuous feeding. C. monitor the patient to see if aspiration continues for another week. D. discontinue tube feeding

Bs

A 32-year-old patient was involved in a motor vehicle accident. He is a ventilator-dependent patient with a tracheostomy tube in place. Because the ventilator controls the respiratory cycle, the patient cannot lengthen exhalations; however, the physician recommends trial therapy for tolerance for swallowing. In preparing for feeding and swallowing, which of the following is recommended? A. Have the patient sit in the bed at a 120-degree angle before occluding the patient's tracheostomy. B. It is preferable to present food to the patient at the end of the exhalation phase of the respiratory cycle. C. It is preferable to present food to the patient at the beginning of the exhalation phase of the respiratory cycle. D. It is too dangerous to present food. Immediately recommend NPO.

C

A 48-year-old stroke patient was misreading words that were irregularly spelled (e.g., Wednesday) but had no problems with non-words that they had never encountered before (e.g., Diggle). This patient probably presents with: A. Functional alexia B. Deep alexia C. Surface alexia D. Phonological alexia

C

A 58-year-old patient who had undergone a total laryngectomy came to an outpatient clinic complaining of coughing when swallowing. The patient was puzzled because he had been told that he would not run the risk of aspiration because there was a physical separation of the gastrointestinal tract and the respiratory tract. The patient had a prosthetic valve in his tracheoesophageal segment to facilitate speaking. The patient said he had understood that the duckbill prosthesis would prevent backflow from the esophagus to the trachea and eliminate aspiration. After completing trial feedings, the speech-language pathologist informed the client that he was coughing because A. the tongue did not have enough rotary movement. B. the size of the bolus was too large. C. there was leakage around the prosthesis. D. he did not time his breathing with each swallow.

C

A 73-year-old client is complaining of issues with regulation of motor movements of the pharynx and lack of sensation of the tongue. What cranial nerve regulates motor movements of the pharynx and sensations of the tongue? A. Cranial Nerve V B. Cranial Nerve VII C. Cranial Nerve IX D. Cranial Nerve XII

C

A 92-year-old patient with dementia is experiencing severe oral-and-pharyngeal dysphagia including aspiration. The clinician would like to teach the patient a swallowing therapy technique, as the family is concerned that he is losing weight. The patient is currently being tube fed and is monitored closely by the hospital staff and dietician. Ethically, which is the only appropriate technique that the clinician should consider? A. Introduce the supraglottic swallow procedure to the patient B. Teach the family members the chin tuck procedure, so that they can feed the patient during visiting hours C. Continue with tube feeding D. Explain to the family that it is okay for the patient to ingest small amounts of food every hour

C

A child is referred to a hospital-based clinician for assessment and treatment. In the child's chart, it is stated that the child has a syndrome caused by spontaneous autosomal dominant mutations. The gene and the locus of this syndrome is FGR2 at 10q25-26. The child has midfacial hypoplasia, an arched and grooved hard palate, and mild mental retardation. What does the child have? A. Trisomy 13 B. Angelman syndrome C. Apert syndrome D. Turner syndrome

C

A client comes to a clinician seeking voice therapy. Derek is a 33-year-old transgender client who has undergone several procedures to become more feminine. Derek now prefers to be called Deena. She is also taking estrogen. Deena is seeking help to speak in a more feminine way but does not know how to go about this. She is also dealing with emotional issues surrounding gender reassignment. In this case, you should A. share with Deena that it is ideal to have voice therapy to teach such feminine communication patterns as increased pitch, increased upward inflection at the ends of utterances, and female body language. B. advise Deena that a combination of voice therapy and counseling will be the best way to sound more feminine and also receive emotional support to deal with gender reassignment issues. C. advise Deena that a combination of speech counseling and voice therapy to teach more feminine pitch levels and communication patterns would be the best course of action and perhaps consult with a physician to determine whether there are surgical options available to change pitch levels. D. tell Deena that various surgical procedures such as thyroplasty are available and that having surgical procedures will be sufficient to help change the voice to sound more feminine.

C

A clinical supervisor is discussing intelligibility with her graduate student clinician. She asks the student, "Which of the following is not a factor that influences intelligibility of an utterance?" A. Frequent abnormality in the clients' speech B. Context in which the communication occurs C. Consistency between the target and its realization D. Loss of phonemic contrasts

C

A clinician is measuring communicative behaviors in a child with a cleft palate. The clinician measures the time intervals during which the speech behaviors selected for observation occurred. What are the methods of measurement called? A. Latency measures B. Self-report measures C. Time sampling measures D. Covert measures

C

A clinician is starting a private practice that focuses primarily on accent training for adult clients who speak English as a foreign language (EFL). Which one of the following is NOT an important consideration in the provision of accent training services? A. have one or two unfamiliar listeners listen to clients' speech samples and determine the percent of unintelligible words B. slowing down a client's rate of speech is often effective in making an immediate improvement in intelligibility C. most EFL clients cannot benefit from the use of the Visi-Pitch because the use of this type of technology is confusing D. Many EFL clients will appreciate doing activities involving reading and discussing information about their countries of origin

C

A clinician measured the number of misarticulations in a child's speech sample in December. She went on winter break and came back 2 weeks later, in January of the new year, rested and refreshed. She decided to listen a second time to the speech sample from the child (which she had recorded in December) and remeasure the number of misarticulations. She did this to establish A. construct validity. B. predictive validity. C. intraobserver or intrajudge reliability. D. interobserver or interjudge reliability.

C

A patient displays persistent aphonia in the absence of any organic cause. The clinician decides to implement behavioral therapy. The following techniques are characteristic of behavioral therapy to treat aphonia, except: A. Masking B. Progressive relaxation leading to reduction in muscle tension C. Chant-talk method D. Yawn-sigh approach

C

A person with otosclerosis often has an audiogram reflecting Carhart's notch,which is A. a specific loss at 2,000 Hz, as indicated by air-conduction testing. B. a specific loss at 4,000 Hz, as indicated by both air- and bone-conduction testing. C. a specific loss at 2,000 Hz, as indicated by bone-conduction testing. D. specific losses at both 2,000 and 4,000 Hz, as indicated by bone-conduction testing

C

A person with otosclerosis often has an audiogram reflecting Carhart's notch,which is A. a specific loss at 2,000 Hz, as indicated by air-conduction testing. B. a specific loss at 4,000 Hz, as indicated by both air- and bone-conduction testing. C. a specific loss at 2,000 Hz, as indicated by bone-conduction testing. D. specific losses at both 2,000 and 4,000 Hz, as indicated by bone-conduction testing.

C

A premature infant was referred for a speech-language evaluation. During the evaluation, the speech-language pathologist noted that the infant had difficulty bringing her hands to her mouth to initiate sucking. While breast feeding, the clinician noted that the infant had bursts and pauses, with about two up-and-down cycles of the jaw per second. This is called A. mouthing. B. sequential sucking. C. non-nutritive sucking. D. nutritive sucking.

C

A researcher teaches a new book reading program to caregivers of children on the autism spectrum and evaluates the children's literacy skills one year later. The researcher's goal is to evaluate whether or not there is a relationship between caregivers' implementation of the program and children's literacy skills. The researcher finds that there is an r =.15 correlational relationship between caregivers' reported implementation of the program and children's literacy skills. The researcher can safely conclude that: A. There is a strong positive correlation between the caregivers' implementation of the program and children's literacy skills. B. There is a strong negative correlation between the caregivers' implementation of the program and children's literacy skills. C. There is no significant relationship between the caregivers' implementation of the program and children's literacy skills D. There is a mildly significant cause-effect relationship between the caregivers' implementation of the program and children's literacy skills.

C

A researcher who was developing a new test of language acquisition in children correlated the scores of children studied with the scores on an established test of known validity. What kind of validity is this? A. Predictive validity B. Content validity C. Concurrent validity D. Construct validity

C

A screening test for hearing that uses a vibrating tuning fork (which is placed on the middle of the forehead) to detect unilateral conductive hearing loss and unilateral sensorineural hearing loss is called the A. chi-squared test B. Mann-Whitney U test C. Weber test D. Rinne test

C

A speech-language pathologist in a rehabilitation setting is treating an adult. The goal of the clinician is to have the adult use a communication device to make sentences with the correct use of various basic grammatical concepts when describing a set of pictures. This is an example of which of the following goals? A. Long-term goal B. Unrealistic goal C. Isolated goal D. Integrated goal

C

A speech-language pathologist is holding a conference with the family of a 16-year-old girl with severe language-learning disabilities. The girl reads at a third-grade level and has been in special education placements since first grade. The speech-language pathologist tells the family, in a kind way, that their goal of their daughter attending medical school is unattainable. The family lashes out in anger against the speech-language pathologist, saying that he is wrong, pessimistic, and negative about their daughter and her abilities. In this situation, the family is utilizing which defense mechanism? A. Repression B. Reaction formation C. Displacement D. Suppression

C

A study that is used to quantify the time and frequency of gastroesophageal reflux into the esophagus is called A. an esophageal reflux monitoring study. B. a GER monitoring study. C. a pH probe or intraluminal pH monitoring study. D. a UGI series

C

Acoustic measurements of voice are becoming extremely popular, especially as a means of evaluating the effectiveness of voice therapy. Sound spectrography, the graphic representation of a sound wave's intensity and frequency as a function of time, yields a spectrogram or picture that reflects: A. measures of jitter and shimmer B. mucosal wave action and optimal pitch C. resonant characteristics of the vocal tract and the harmonic nature of the glottal sound source D. fundamental frequency

C

Advances in genetic research indicate that a mutation in four genes cumulatively account for about 20% of unrelated cases of persistent stuttering. When conducting the research, the investigators carefully matched those who stuttered and controls geographically but not genetically; therefore, subtle differences in ancestry were not taken into account. Findings, however, indicate that mutations in the following four genes may play a part in persistent stuttering. A. BRCA1, BRCA2, CHEK2, TP53 B. HTT, TP53, TNF, EGFR C. GNPTAB, GNPTG, NAGPA, AP4E1 D. VEGFA, APOE, IL6, TGFB1

C

An 86-year-old patient has been diagnosed with a delayed swallowing reflex post CVA. The modified barium swallow study indicates that there is pooling in the vallecula until the swallowing reflex has been triggered. The clinician decides to give the patient a mechanical soft bolus and ask her to swallow while putting her head down (chin tuck). This technique will result in A. increasing the change of aspiration. B. narrowing or closure of the vallecula space. C. narrowing of the airway entrance, as well as pushing the epiglottis posteriorly. D. pushing the tongue base forward.

C

An attending physician refers a patient for swallowing therapy. The patient has a tracheostomy tube in place. Which of the following is recommended while conducting therapy? A. There is no need to suction after feeding, as there is no chance of aspiration. B. Feed the patient when the cuff is inflated. C. Do not feed when the cuff is inflated. D. Recommend immediate NG-tube feedings.

C

An audiologist is conducting a hearing test and sends an acoustic signal through a headphone at a level that is strong enough to obscure the tone heard in the opposite ear. Which of the following statements is not true about masking? A. Masking allows the clinician to eliminate the non-test ear from participation while measuring hearing thresholds for the test ear. B. The masking noise in the non-test ear must be enough to make any sound crossing the skull from the test ear inaudible. C. For testing hearing by bone conduction, masking is needed whenever the difference between the air-bone gap is 60 dB or more. D. The plateau method is a procedure used to determine how much masking is sufficient.

C

Because of improved treatment for HIV infection, patients are now living longer. However, as they live longer, some are prone to dementia due to that infection. Select the statement that is true of the AIDS dementia complex. A. Onset is sudden, and deterioration is slow. The HIV infection itself is never the cause; opportunistic brain infections cause deterioration. B. ith sudden onset, the patients show rapid and severe deterioration in their speech and language skills. This type of dementia is cortical. C. The onset is slow, but deterioration is rapid in the final stages; tremors, seizures, gait problems, facial nerve paralysis, incontinence, and confusion, depression, hallucinations, delusions, and mutism in the final stage characterize the AIDS dementia complex. D. Unlike other forms of dementia, AIDS dementia complex is free from problems of memory, concentration and attention, apathy, loss of interest in work; thinking is well preserved until the final stage.

C

Before a surgeon performs a Type V total glossectomy, she mentions to the speech-language pathology that all of the mobile tongue including the base of the tongue will be transected at the level of the valleculae. She also informs the clinician that the _______ nerve will be transected. A. Cranial nerve X, vagus nerve B. Cranial nerve IX, glossopharyngeal nerve C. Cranial nerve XII, hypoglossal nerve D. Cranial nerve VII, facial nerve

C

Communication deficits are common in TBI. Which disorder occurs in one third of the TBI population? A. Aphasia B. Apraxia C. Dysarthria D. Dementia

C

Constrictions in the vocal tract will raise or lower the formant frequency. How is the first formant frequency, which is also known as F1, lowered? A. A constriction near the velum B. A constriction near a volume velocity maximum C. A constriction near a pressure maximum D. A constriction in the pharynx

C

Constrictions in the vocal tract will raise or lower the formant frequency. How is the first formant frequency, which is also known as F1, raised? A. Any constriction in the oral cavity B. A constriction in the oral cavity near a volume velocity maximum C. A constriction in the pharynx D. A constriction in the oral cavity near a pressure minimum

C

Damage to the _______ gyrus often results in Gerstmann syndrome which may result in alexia, agnosia, acalculia, left-right confusion, and agraphia. A. cingulate B. fornicate C. angular D. lingual

C

Darren, a client with cerebral palsy who was just added to your caseload, presents with uncontrollable movements that are rapid and jerky. You observe that Darren is experiencing trouble controlling movement in his hands, arms, feet, and legs. It appears that his tone changes throughout the day. What type of cerebral palsy does Darren have? A. Spastic B. Ataxic C. Dyskinetic D. Mixed

C

Debbie, a 42-year-old, was diagnosed with laryngeal cancer and has been noticing changes in her voice. Which of the following is the primary voice symptom of laryngeal cancer? A. diplophonic B. breathiness C. hoarseness D. strained

C

Evaluation refers to the process of arriving at a diagnosis. Clinicians perform evaluation tasks with two major goals in mind. All of the following statements are true regarding evaluation except: A. Informal probes, trial therapy tasks, and generalization data are part of an evaluation. B. We evaluate to arrive at a good understanding or diagnosis of a client's problem. C. Therapy with a client typically begins after arriving at a firm diagnosis. D. A major reason to perform evaluation activities is to monitor the client's progress in treatment.

C

Following a stroke, a 65-year old patient displays the following symptoms. She is unable to identify unfamiliar words and is having difficulty sounding out written words. When reading a passage, the patient reads the word "horse" instead of the word "donkey." In terms of treatment planning, it appears that the patient has __________. You inform the family that your therapy will focus on phonological treatment (sublexical skills) and maximizing spelling performance (interactive treatment to train lexical and sublexical information). A. pure alexia B. alexia without agraphia C. deep alexia D. surface alexia

C

Imitation is important for the growth of language development in infants. Before assessing an infant with a language disorder, it is important to understand the imitation that occurred between the caregiver and the infant. How long immediately after birth can an infant imitate? A. 3 weeks B. 1 month C. a few hours D. 1 week

C

In the following stage of stuttering modification therapy, clients are asked to confront the disorder, review core behaviors, raise the threshold of fixations and oscillations, and judge listener actions: A. modification B. stabilization C. desensitization D. identification

C

In the past 20 years, there have been changes to the Medicare statute that affect speech-language pathologists. Under which change was there a 1.5% increase in the Medicare payment schedule for services paid? A. The Medicare, Medicaid, and SCHIP Balanced Budget Refinement Act of 1999 (BBRA) B. The Medicare, Medicaid, and SCHIP Benefits Improvement and Protection Act of 2000 (BIPA) C. The Medicare Prescription Drug, Improvement, and Modernization Act of 2003 (MMA) D. The Medicare Percentage Based Payment Act of 1998

C

In the scientific method, what is the experiment-first-and-explain-later approach? A. Deductive method B. Null hypothesis method C. Inductive method D. Alternative hypothesis method

C

In your job in a NICU, you find that many of the babies have feeding difficulties. Which one of the following statements is false regarding medically fragile babies with feeding difficulties? A. They have problems with oral-motor development. B. They often need to be fed through nasogastric tubes. C. They can generally breastfeed easily. D. To be fed orally, they must be at least 35 weeks old.

C

Intracranial arteritis is a vascular etiology that is rarely or never a possible cause for what type of motor speech disorder? A. Spastic dysarthria B. Ataxic dysarthria C. Hypokinetic dysarthria D. Apraxia of speech

C

Jason, a 28-year-old patient, was skiing without a helmet and crashed head first into a tree. He subsequently experienced a traumatic brain injury. You need to develop a plan of treatment for Jason that is functional throughout his daily routine. Jason notes that he was previously an accountant and would like to take steps toward returning to his career. Which of the following would not be appropriate to initially focus on treatment during his treatment session. A. increasing his memory for daily routines B. integrating work or school related words, phrases, and narratives for communication C. increasing his written/oral grammatical correctness D. improving his communication attention

C

Natalie is 4-year-old with cerebral palsy and has been diagnosed with gastroesophageal reflux. She frequently throws up and cannot retain nourishment. The physician diagnoses Natalie with a weak or hypersensitive: A. digastricus. B. palatoglossus. C. esophageal sphincter. D. epiglottis.

C

Nicole, a voice major, is having problems raising the pitch of her voice. Her speech-language pathologist recommends that she lengthen and tense her vocal folds to increase her pitch. Which muscle is involved in achieving this goal? A. Transverse arytenoids B. Oblique arytenoids C. Cricothyroid D. Thyroarytenoid

C

Pushing/pulling exercises are behavioral approaches that can be useful treatment techniques when treating which type of dysarthria? A. hyperkinetic B. spastic C. flaccid D. ataxic

C

Remy is a Tagalog-speaking second grade boy who is referred to you by his teacher for a possible language impairment. She is concerned because Remy seems to interact well socially in the classroom and is fluent in informal, conversational English. However, Remy is somewhat behind his monolingual English-speaking peers in science and math. The teacher wonders if Remy needs speech-language therapy to boost his science and math skills so that his performance in these areas is commensurate with that of his peers. You tell her that: A. There is a strong possibility of a language impairment, and that you will assess Remy as soon as possible B. You will refer Remy to the resource specialist and psychologist, because there is a good chance he has a learning disability C. Remy is probably a typically-developing student who has developed adequate conversational informal language fluency and needs more time to develop formal academic language fluency D. Remy will not make progress in a typical classroom, and needs to be sent to a special school where he can be in a small classroom with children with documented special needs

C

Researchers often use cross-sectional studies to study children's development in various areas. What is a difficulty with cross-sectional studies? A. The same subjects are studied over time, and this is expensive, time consuming, and difficult because subjects might drop out of the study. B. The investigator is examining data already on file to answer questions about children in various age groups, and that data might not be reliable. C. Observations are made of differences between subjects of different ages to generalize about developmental changes that would occur within subjects as they mature. D. The total age span of children to be studied is divided into several overlapping age spans, and it is difficult to follow subjects from the lower to the upper end of each age span.

C

Resonance is a property of the vocal tract. Resonance exists whether it is energized or not. What kind of effect does resonance have on speech? A. Resonance does not modify the energy of a speech signal. B. Resonance adds energy to the speech signal. C. Resonance does not add energy to the speech signal. D. It is still unknown.

C

Rhianna, a 22-year-old female, is 6-days post-traumatic brain injury (TBI) that she suffered from a motor vehicle accident. She has not history of substance abuse and is considered to be in good health prior to TBI. She emerged from a come 36 hours ago and is not at Rachos Los Amigos Level--IV. She is confused and agitated. A neurologist gives you an hour to evaluate Rhianna and to provide recommendations. What information would contribute to a fair-good prognosis? A. Younger patients have higher mortality rates B. History of substance abuse C. Comatose to Level IV on RLAS in 36-hour period D. Six days comatose

C

Sarah, the mother of 18-month old toddler Brandon, has taken time off her job as an engineer to be with Brandon full time at home. Sarah is very conscientious about Brandon's speech and language development, and brings him to you for an evaluation "just to make sure he's on track." Which of the following would Brandon be expected to demonstrate at 18 months of age if he is developing in a typical fashion? A. 20-30 expressive vocabulary words, mostly consisting of nouns B. establishment of joint reference and 10-15 expressive vocabulary words C. an expressive vocabulary of 50 words as well as beginning to put 2 words together D. primary use of 3-4 word responses and use of "and" to join sentences

C

Select the statement that is not true. A. Dysarthria and Broca's aphasia may coexist. B. Excessive or even stress on syllables is a part of ataxic dysarthria. C. Roughly 94% of Parkinson's patients have hypokinetic dysarthria. D. Spastic-ataxic and flaccid-spastic are frequently mixed in the mixed variety of dysarthria.

C

Shirley, a 60-year-old patient, was admitted to the hospital with a diagnosis of severe oropharyngeal dysphagia. There were no overt signs or symptoms of aspiration at bedside; however, Shirley had weak and decreased laryngeal elevation and excursion during her swallow as per palpation. The clinician noticed a change in Shirley's respiratory statue and after consulting with her respiratory therapist, decided to request a videofluroscopic swallow study. Results indicated residue throughout the oral cavity as well as in the valleculae and pyriform sinuses that was partially cleared using compensatory strategies. The report also indicated inconsistent silent aspiration, a delayed swallow, and severely decreased laryngeal elevation and excursion. The clinician decided to make the patent NPO without PO trials to prevent aspiration until swallow strength and function increased. Which of the following would not be appropriate treatment for Shirley? A. Thermal stimulation B. Effortful swallow C. Practicing liquid swallow after stimulation D. Mendelsohn maneuver

C

Siobhan, a 74-year-old female, is a new resident at the nursing home. After reading her case history, you learn that she recently has had a history of multiple strokes. In her case history, it states that her cognitive impairments coincided with her strokes. She is disoriented, confused, has trouble speaking or understanding speech, and has vision loss. Other symptoms include uncontrolled laughing and crying, problems with judgment and planning, and a reduced ability to pay attention. Her daughter reports that Siobhan appears to display symptoms of depression, noting that "ever since the strokes, Mom has a difficult time with simply getting up in the morning and she just doesn't seem to enjoy being around her family like before." In person, Siobhan shows loss of motivation and attention during various tasks. She appears to have some memory loss, but primarily struggles with executive functioning. Siobhan also presents with small-step gait and unsteadiness. She is prone to falls and is required to walk with a cane. Her regular nurses report that she has urinary urgency not explained by urologic disease along with emotional incontinence. Due to concerns expressed by her family, Siobhan is being evaluated to determine the presence of dementia. All of the following are probable etiologic subtypes for Siobhan's dementia, except: A. Hypoperfusion in border zones and granular cortical atrophy B. Multilacunar state C. Excess amount of neurofibrillary tangles D. Postischemic encephalopathy

C

Speech is a dynamic process requiring precisely coordinated articulatory movements for the sequencing of sounds to ultimately produce words. What is common among children with cerebral palsy? A. Apraxia B. Flaccidity C. Speech impairment D. Hyponasality

C

Speech-language pathologists in educational settings need to ask themselves questions that will guide curriculum-based language communication assessments. One question is "What communication abilities and tactics might the child obtain to become successful?" What can the clinician to do answer this question? A. Investigate how the student contributes to conversations. B. Observe the student in the classroom. C. Investigate what kinds of scaffolding might help the student. D. Collaborate with other educators to problem solve

C

Stuttering often occurs most commonly on all of the following situations, except: A.Content words B. The first sound or syllable of a word C. Function words D. Longer words

C

The lowest intensity of a sound that will stimulate the auditory system is called A. sound pressure level. B. decibel. C. hearing level. D. pitch.

C

The most basic requirement for the caregiver-infant attachment is ___________________ between caregiver and infant. A. Smiles B. Touch C. Recognition D. Shelter

C

The most common symptom of cerebral palsy is: A. Ataxic dysarthria B. Bradykinesia C. Spasticity D. Hemiballism

C

The two muscles in the middle ear that dampen the vibrations of the tympanic membrane and the ossicular chain are the A. levator tympani and stapedius muscle B. auditory meatus and elevator tympani C. tensor tympani and stapedius muscle D. buccinator tympani and risourius

C

There are several aspects involved in studying language. These include using language as a social tool, a learned behavior, and/or a linguistic means of communication. Which of the following defines language as a learned behavior? A. Verbal language that is learned through trial and error B. Verbal language that is learned through imitation C. Verbal language that is learned through social consequence D. Verbal language that is learned through intentions

C

There are two broad levels of human communication: verbal communication and nonverbal communication. Verbal communication uses words as symbols to exchange ideas. Nonverbal communication includes many behaviors that communicate with or without the associated production of symbols. Which of the following is not an example of verbal communication? A. American Sign Language B. Written language C. Extralinguistic communication D. Pictographic communication

C

There are various types of surgical and prosthetic voice rehabilitation techniques for total laryngectomy patients. Which of the following is the most successful surgical prosthetic procedure? A. electrolarynx B. esophageal speech C. tracheoesophageal puncture procedure D. staffieri neoglottis procedure

C

Unawareness is common among patients with traumatic brain injuries and can affect intervention on task performance in all of the following except when patients ____________________. A. Begin tasks without planning or setting goals B. Focus on irrelevant aspects of tasks C. Recognize errors D. Do not spontaneously initiate strategies

C

Van Riper's approach to treating stuttering included all of the following except: A. Desensitization B. Modification C. Continuous phonation D. Motivation

C

Various arteries help supply blood to the face and the brain. Neurogenic communication disorders are associated with interrupted blood supply to the brain. Of the following statements about the arteries that supply blood to the brain, which one is correct? A. The internal carotid artery supplies the muscles of the face. B. Damage to the external carotid artery causes aphasia. C. Broca's area and Wernicke's area are supplied by the middle cerebral artery. D. If an artery below the circle of Willis is blocked, brain damage is maximal.

C

What are the two forces needed for vibration? A. Mass and elasticity B. Mass and inertia C. Inertia and elasticity D. Weight and inertia

C

What contributes to moderate amounts of physiologic recovery for patients with moderate traumatic brain injury? A. Surgery B. Medication C. Neuroplasticity D. Rehabilitation

C

What does the cover-body theory of phonation state? A. The superficial layer and much of the intermediate layer of the lamina propria vibrate as a "cover" on a relatively stationary "body" that is made up of the remainder of the intermediate layer, the deep layer, and the thyroarytenoid (TA) muscle. B. The epithelium, the superficial layer of the lamina propria, and much of the intermediate layer of the lamina propria vibrate as a "cover" on a relatively stationary "body" that is made up of the remainder of the intermediate layer and the deep layer. C. The epithelium, the superficial layer of the lamina propria, and much of the intermediate layer of the lamina propria vibrate as a "cover" on a relatively stationary "body" that is made up of the remainder of the intermediate layer, the deep layer, and the TA muscle. D. The epithelium, the deep layer of the lamina propria, and much of the superficial layer of the lamina propria vibrate as a "cover" on a relatively stationary "body" that is made up of the remainder of the superficial layer, the deep layer, and the TA muscle.

C

What does the intervention for comatose or semi-comatose primarily consist of? A. Visual processing B. Motor reflexes C. Sensory stimulation D. Memory impairments

C

What is injected into paralyzed vocal folds to increase bulk and possibly increase adduction? A. Steroids B. Antibiotics C. Collagen D. Stem cells

C

What is one of the reasons that production of speech sounds is restricted in infants? A. When a human is born, an inadequate larynx-to-lung ratio is present. The lungs are too small to create enough subglottal pressure for the infant's larynx; therefore, phonation is unable to occur. B. Infants do not have any teeth, so their phoneme inventory is limited to sounds other than dentals. C. An infant's tongue completely fills the oral cavity and the pharyngeal cavity is used for sucking and swallowing; therefore, the larynx and vocal tract are unable to perform any secondary functions. D. Infants breathe at a slow rate yet are unable to completely expand their rib cage. This means that infants are unable to create enough positive and negative pressure to perform the task of respiration with enough power to create phonation.

C

What is the difference between the vocal folds of men and women? A. The vocal folds of men are longer and have more mass than those of women, resulting in a higher fundamental frequency. B. The vocal folds of men are shorter and have less mass than those of women, resulting in a lower fundamental frequency. C. The vocal folds of men are longer and have more mass than those of women, resulting in a lower fundamental frequency. D. The vocal folds of women are shorter and have less mass than those of men, resulting in a lower fundamental frequency.

C

What is the primary clinical task for a speech-language pathologist when assessing a client with cleft palate? A. Infer the effects of structural deviations on the speech behavior observed B. Compare data to developmental norms C. Assess the child's speech sound system D. Confirm a diagnosis

C

What reinforcers include secondary and social reinforcers? A. Primary reinforcers B. Unconditioned reinforcers C. Conditioned reinforcers D. Decommissioned reinforcers

C

What should be the deciding factor(s) for a developmentally based intervention regarding speech sound therapy? A. Mastery charts B. Articulation tests C. Mastery charts and articulation tests D. Mastery charts, articulation tests, and state standards

C

What would you call an experimental design involving one or a few subjects? A. Case study design B. Ex post facto design C. Single-subject experimental design D. Single correlational design

C

When analyzing spectrograms on a spectrograph, there are different types of spectrograms that allow you to analyze your sample to the best of its ability. These different types of spectrograms are called wideband spectrograms and narrowband spectrograms. Which of the following characteristics is expressed better in a narrowband spectrogram? A. Short time window B. Good for measuring formant frequencies C. Good for showing and measuring harmonics D. Good for showing changes in articulatory movement

C

When analyzing spectrograms on a spectrograph, there are different types of spectrograms that allow you to analyze your sample to the best of its ability. These different types of spectrograms are called wideband spectrograms and narrowband spectrograms. Which of the following characteristics is expressed better in a narrowband spectrogram? A. Short time window B. Good for measuring formant frequencies C. Good for showing and measuring harmonics D. Good for showing changes in articulatory movements

C

When completing a language evaluation, it is important for a speech-language pathologist to understand the basic elements of language. The five components of language include semantics, syntax, morphology, pragmatics, and phonology. Which component of language refers to the rules governing grammatical constellation of language units? A. Phonologic B. Morphologic C. Syntactic D. Semantic

C

When completing a language evaluation, it is important for a speech-language pathologist to understand the basic elements of language. The five components of language include semantics, syntax, morphology, pragmatics, and phonology. Which component of language refers to the rules governing grammatical constellation of language units? A.Phonologic B. Morphologic C. Syntactic D. Semantic

C

When considering an infant's readiness for oral feeding, all of the following must be taken into account, except: A. Severity of medical condition B. Gestational age C. Chronological age D. Behavioral state

C

When differentiating articulation from phonological disorders, what is the purpose of summarizing the collapse of phonetic contrasts? A. To determine any sound or sound blends that characterize different phonemes B. To determine what sounds are inconsistently used C. To determine any substitutions that characterize more than one target phoneme D. To determine what sounds are consistently used

C

When evaluating the voice of an individual with laryngeal cancer, what is the most valuable tool in the assessment of a voice disorder? A. case history B. past medical history C. patient interview D. treatment protocol

C

When focusing on caregiver-infant attachment, it is important that the dyad relationship is properly assessed before exploring treatment options. From the choices provided, what would best constitute a dyad? A. Non-biological B. Emotional C. Two-way D. Biological

C

When planning treatment for a client with cleft palate or related velopharyngeal inadequacy, it is necessary to assess: A. Clinical procedures only B. Instrumental procedures only C. Clinical and instrumental procedures D. Surgical procedures only

C

When treating a child with cerebral palsy, it is important to work with a team of specialists. Which specialist would be the least likely to work with a child with cerebral palsy? A. Physical therapist B. Speech-language pathologist C. Podiatrist D. Teacher

C

When treating apraxia of speech (AOS), some clinicians teach clients to shape articulatory productions and capitalize on productions that are automatic. This technique includes the use of phoneme drills. What is this referred to as? A. Voluntary control of involuntary utterances B. Prompts for reconstructing oral muscular targets C. Darley, Aronson, and Brown's (1975) procedure for AOS D. Eight-step continuum treatment for AOS

C

Which area of the brain connects Broca's area with Wernicke's area? A. corpus callosum B. choroid plexus C. arcuate fasciculus D. corona radiata

C

Which is not a primary objective of assessment for patients with disordered consciousness? A. Determine the patient's level of consciousness. B. Get a sense of the nature and severity of the patient's injuries. C. Consider environmental conditions. D. Estimate the patient's physical, behavioral, and cognitive recovery between time of injury and the time of assessment.

C

Which is not an appropriate treatment method when treating articulation and phonological disorders for a client with cerebral palsy? A. Assess the child's specific sound errors and error patterns. B. Evaluate the compensatory articulatory postures the child uses. C. Generalize techniques that have worked for previous clients with cerebral palsy, and not individualizing therapy. D. Teach specific phonemes or the classes of phonemes based on distinctive features or phonological patterns.

C

Which is not true about behavioral and cognitive recovery for patients with a traumatic brain injury (TBI)? A. Age is the most important patient-related variable for predicting recovery for TBI. B. Patients with TBI typically progress through a fairly predictable sequence of stages as they recover. C. Predictability of the permanent level of impairment of patients with TBI D. Recovery from TBIs often follows a stair-step pattern, alternating between intervals of little or no change to intervals of rapid improvement.

C

Which of the following assessments is used to identify the motor, verbal, and eye opening response? A. Traumatic Brain Injury Scale B. TBI Checklist C. Glasgow Coma Scale D. Ranchos Los Amigos Scale

C

Which of the following is a limitation of standardized speech-language tests? A. Generally exhaustive time required to administer them B. Lack of statistical norms C. Inadequate participant and response sampling D. Unnecessarily extensive testing of each individual skill sampled in the test

C

Which of the following is important when treating language disorders for a child with cerebral palsy? A. Teaching the parents techniques learned from the team of specialists working with their child B. Assessing the child's language and prescribing medications that are appropriate in improving speech C. Using and modifying treatment procedures to suit the individual child with cerebral palsy D. Considering the child's rate of speech and how it affects his or her visual coordination

C

Which of the following is not true about the Rancho Los Amigos Scale of Cognitive Levels (RLAS) and the Ranchos Los Amigo Scale-Revised (RLAS-R)? A. they were designed to include a more comprehensive estimate of the cognitive behavioral characteristics of the patients with traumatic brain injuries B. they measure patients with brain injuries according to their arousal, responsiveness, restlessness, attention, memory , executive ability, and level of independence C. the length of time that a patient remains at each level of the RLAS is related to the eventual outcome of the patient's recovery D. the five highest levels of the RLAS are more sensitive to language impairments than the five lowest levels

C

Which of the following is true about myopathies? A. Myopathies have an affinity for lower motor neuron (LMN) cell bodies. B. Myopathies reflects mutations in genes that influence the suppression of tumors. C. Myopathies are not associated with sensory deficits or central nervous system (CNS) pathology. D. Myopathies are a group of disorders that involves regeneration of the stretch reflex

C

Which of the following is true about myopathies? A. Myopathies have an affinity for lower motor neuron (LMN) cell bodies. B. Myopathies reflects mutations in genes that influence the suppression of tumors. C. Myopathies are not associated with sensory deficits or central nervous system (CNS) pathology. D. Myopathies are a group of disorders that involves regeneration of the stretch reflex.

C

Which of the following statements is true about cerebral palsy? A. All children with cerebral palsy have an intellectual disability. B. Cerebral palsy does not occur later in life. C. Cerebral palsy is a nonprogressive, nondegenerative disease. D. Cerebral palsy is curable

C

Which one of the following word pairs contains homophenous sounds? A. Tin-kin B. Lace-race C. Man-pan D. Veil-whale

C

Which type of communication is achieved through the assistance of external devices? A. Unaided communication B. Nonverbal communication C. Aided communication D. Alternative communication

C

While evaluating the efficacy of a treatment procedure for aphasia, an investigator made sure that the clients in the experimental and control groups did not receive any other form of therapy during the course of the experimental study. This was done to A. increase the amount of improvement in the clients. B. eliminate scheduling conflicts. C. rule out the influence of confounding variables. D. ensure that the Hawthorne effect was not operating

C

With regard to human communication, which of the following viewpoints studies the process of sending and receiving messages? A. Behavioral B. Analytical C. Theoretical D. Linguistic

C

You are asked to design a treatment program for young children who stutter, including preschoolers. Among the following choices, which would you most likely select A. Delayed auditory feedback B. Masking noise C. Fluency reinforcement D. Fluent stuttering

C

You are developing a language treatment program for a child diagnosed with autism. In your treatment program, you include which of the following main and initial skills? A. Morphologic language skills B. Syntactic language skills C. Pragmatic language skills, including social appropriateness D. Increased verbalizations on topics of interest for the child

C

You are doing a fluency evaluation on a 5-year-old boy. You determine he is stuttering on about 25% of syllables and his dysfluencies average 3 to 4 seconds in duration. You notice __________ as a core behavior of his stuttering. He also has facial grimacing and distracting movements. A. whole word repetitions B. interjections C. blocks D. secondaries

C

You are evaluating a 3-year-old child whose parents have concerns about his social aspects of communication, frequent echolalia, and perseverations. His parents mentioned that he frequently talks to himself, has anxiety, and displays hyperactive behavior. These concerns are evident throughout your evaluation. You notice that he has difficulty attending to tasks and has limited eye contact. Upon observation, his facial features are characterized by a high forehead, large jaw, and a poorly formed pinna. Following your evaluation, you refer this child and his family to a geneticist to rule out the possibility of a genetic syndrome. The patient most likely presents with: A. Down syndrome B. Pierre-Robin syndrome C. Fragile X syndrome D. Prader-Willi syndrome

C

You are evaluating a patient suspected of severe apraxia of speech (AOS). Which of the following is not a characteristic of severe AOS when distinguishing it from less severe forms? A. Error responses may approximate the target if stimuli are chosen carefully B. Limited repertoire of speech sounds C. Increased variability of articulatory characteristics D. Automatic speech may not be better than volitional speech

C

You are providing services to Mr. W., a patient with Parkinson's disease and consequent dysarthria, to help him sound more intelligible. You can expect that you will need to address challenges related to A. even and consistent breakdowns in articulation. B. impaired syntactic structures. C. forced inspirations and expirations that interrupt speech. D. an increased rate of speech under pressure.

C

You are screening the speech of a third-grade Spanish-speaking student named Araceli. Which of the following would not be typical for Araceli based on the influence of her primary language of Spanish? A. "My sister Yulie [Julie] is coming." B. "I like berry much my teacher." C. "My mommy and me be goin' shoppin' later." D. "My friends always say 'ello [hello] to me."

C

You are working with a child who stutters and are training the parents in indirect therapy. Which of the following is most appropriate? A. Response cost B. Fluent stuttering method C. Using indirect prompts instead of direct questions D. Fluency reinforcement method

C

You have a new client who can accurately produce the /t/ sound in the initial and final position of syllables, and in the initial, medial, and final positions of words. The client has inaccuracies with producing /t/ in phrases. According to the topographic levels of treatment, where should you begin treatment? A. Syllables B. Words C. Phrases D. Sentences that are imitated

C

You have been asked to give an in-service to a group of students who wish to eventually specialize in service delivery to children with cleft palates and their families. The students want to know detailed information about in utero development of the hard and soft palates (among other things). You can accurately tell them that in utero the hard palate fuses between which developmental ages? A. 1-2 weeks B. 4-6 weeks C. 8-9 weeks D. 10-12 weeks

C

You want to evaluate your client's ability to make rapidly alternating speech movements by using diadochokinesis (DDK) syllable rates. You evaluate how many repetitions of /pʌ/ the client can produce in 15 seconds. All of the following are true regarding DDK syllable rates except: A. There are two primary ways to obtain these measures. B. Before obtaining a client's rate, you should provide instructions for the tasks, model the target behaviors, and give the client time to practice. C. If the client stops or slows down intentionally before the allotted time is expired, you should take note of this, but the task does not need to be repeated. D. The DDK rate can help determine the severity of a communication disorder, its cause, and the best course of treatment.

C

_____________ is a central cognitive principle. This principle involves scheme, which defines the way humans organize behavior into identifiable patterns. A. Equilibrium B. Imitation C. Organization D. Adaptation

C

___________________ memory consists of remembering facts and events, and refers to the memories that can be consciously recalled? A. Procedural B. Short Term C. Declarative D. Implicit

C

After an articulation disorder has been diagnosed, the most appropriate sequence of therapy should be: A. Correct Production of the Sound in Specific Contexts B. Earlier Developmental Sounds C. Stimulability D. Sounds that Affect Intelligibility

C, A, B, D

The classification of unaided symbols can be separated into which of the following two categories? A. Object based B. Arbitrary logogaphs C. Linguistic D. Alphabet-based symbols E. Arbitrary shapes F. Nonlinguistic

C, F

The anterior cerebral artery supplies blood to the

Corpus callosum and basal ganglia

The cranial nerve that innervates the larynx and also innervates the levator palatini, palatoglossus, and the palatopharyngeus muscles is the

Cranial nerve X, the vagus nerve

A 19-year-old student was airlifted to a hospital following a car accident. He had difficulty breathing, as the steering wheel had compressed his chest. H had been immediately intubed by the paramedics at the scene of the accident to establish an airway. Three days after the accident, he was still intubated, as he continued to have difficulty breathing on his own. What would be the most appropriate and ethical course for feeding the patient? A. To introduce the Mendelssohn maneuver B. To introduce the supraglottic swallowing procedure C. To provide the patient with one glass of water for nourishment D. To disallow any swallowing therapy until the intubation is removed

D

A 72-year-old patient was diagnosed with apraxia of speech post CVA. The clinician instructed the patient to "watch and listen to me" and then "say it with me." The aim of the treatment approach was to use simultaneous practice to bring to consciousness the look and sound of the target speech output. This is an example of A. metrical pacing B. metronomic pacing C. articulatory cueing D. integral practice

D

A 72-year-old patient was diagnosed with apraxia of speech post CVA. The clinician instructed the patient to "watch and listen to me" and then "say it with me." The aim of the treatment approach was to use simultaneous practice to bring to consciousness the look and sound of the target speech output. This is an example of A. metrical pacing. B. metronomic pacing. C. articulatory cueing. D. integral practice.

D

A 9-year-old was seen for an evaluation because of a major dysfunction in his gastrointestinal tract. The child had short bowel syndrome, resulting in the removal of a major part of the intestines. The gastroenterologist most likely would recommend A. a jejunostomy tube (j-tube) B. a duodenal tube (duo-tube) C. a gastrojejunal tube (GJ-tube) D. total parenteral nutrition (TPN)

D

A Cambodian child is referred to you because the teacher "can't understand a word he says." In the course of your speech-language screening, you record the following utterances. Which might be typical of an articulation disorder, not a difference? A. "The funny circus crown had red nose." B. "I won't ste on your toes." C. "Please hand me the block [black] one." D. "I am derry [very] appy [happy] to meet your tids [kids]."

D

A child comes to your clinic with her mother for articulation and language therapy. The mother tells you that her daughter has Hurler's syndrome. What causes Hurler's syndrome? A. An expanded number of CGG nucleic acid repeats on a specific gene on one of the distal ends of the Y chromosome B. A spontaneous autosomal dominant mutation of FGR2 at 10q25-26 C. Autosomal dominant inheritance and deletion in the region of the long arm of chromosome 15 (15q11-15q13) D. Autosomal recessive deficiency of X-L iduronidase

D

A client who stutters mentions to his speech-language pathologist that his social life is limited. He states, "No one will talk to me because I stutter." This is an example of the common defense mechanism known as A. displacement. B. projection. C. repression. D. rationalization.

D

A clinical supervisor is quizzing her graduate student clinician during a break from seeing patients. At their rehabilitation center, they see many patients with cerebral palsy. She asks the student what the most appropriate characteristics would be if congenital or neurodevelopmental motor speech disorders are associated with cerebral palsy. What is the correct answer? A. Apraxia of speech B. Aphasia C. Dysphagia D. Dysarthria

D

A free morpheme can stand alone and still carry meaning. A bound morpheme must be attached to other (free) morphemes to carry meaning. Which of the following is an example of a word containing both a free and a bound morpheme? A. Mess B. Happy C. Waterfall D. Replay

D

A high school teacher refers a Mandarin-speaking 16-year-old to you for an evaluation. The student and his family came to the United States 2 years ago from China. The teacher says that the student does well academically, but she shares that she has difficulty understanding him when he speaks. When you screen the student, you find some articulation and language differences. Which one of the following would not be predictable based on the student's first language of Mandarin? A. Substitutions of t/th (e.g., tin/thin) B. Epenthesis in words with consonant blends C. Confusions of /r/ and /l/ D. Substitutions of f/th (e.g., fick/thick)

D

A kindergarten teacher refers Tomiko to you for a speech-language screening. Tomiko's first language is Korean, and she has been exposed to English for 8 months in school. The teacher is concerned because he thinks that Tomiko "has a speech problem." Which one of the following patterns (in English) would you not expect to find in a student who speaks Korean? A. Substitution of a/ae (e.g., bock/back) B. Final consonant deletion (e.g., be-/bed) C. Confusion of r/l (e.g., glow/grow, lay/ray) D. Substitution of t/k (e.g., tea/key)

D

A new client is placed on your caseload who is transsexual and is currently transitioning from male to female. This client received a thyroplasty and longitudinal incision of the vocal folds to achieve a higher-pitched voice, as the typical characteristics of a "female voice" are important to her. Voice therapy, however, was still recommended following surgery. The following would be important to address in therapy, except: A. Build vocal stamina and endurance to decrease vocal fatigue B. Use a greater number of rising pitch inflections at the ends of utterances C. Speak with greater articulatory precision, a softer voice, and more models D. Position the tongue more posteriorly during speech to create a resonance that is considered to be more feminine

D

A physician refers a 50-year-old male patient with dementia to you for assessment and treatment. The referring physician suggests the strong possibility of dense intracellular formation in the neuronal cytoplasm and ballooned and inflated neurons. Your assessment reveals that the patient has had a progressive loss of vocabulary, paraphasia, circumlocution, and dominant language problems, with somewhat better preserved memory and orientation. The most likely diagnosis you would make on this patient is A. dementia of the Alzheimer's type. B. dementia associated with Parkinson's disease. C. dementia associated with Huntington's disease. D. frontotemporal dementia associated with Pick's disease.

D

A physician refers a 50-year-old male patient with dementia to you for assessment and treatment. The referring physician suggests the strong possibility of dense intracellular formation in the neuronal cytoplasm and ballooned and inflated neurons. Your assessment reveals that the patient has had a progressive loss of vocabulary, paraphasia, circumlocution, and dominant language problems, with somewhat preserved memory and orientation. The most likely diagnosis you would make on this patient is A. dementia of the Alzheimer's type B. dementia associated with Parkinson's disease C. dementia associated with Huntington's disease D. frontotemporal dementia associated with Pick's disease

D

A researcher is interested in completing a single-subject design experiment to study the additive and subtractive effects of individual components of treatment rather than the comparison of two treatments. He decides to use an interaction design to study the interactive effects of two or more variables. The researcher is also interested in examining the effects of both variables alone and in combination. He is interested in isolating the components that are effective to any extent from those that are not at all effective and controls for phase lengths during the treatment. He also counterbalances the order of treatment in the experiment. The most appropriate design for this experiment would be A. ABAB. B. BAB. C. A-B-A-BC. D. A-B-BC-B-BC.

D

A researcher was using the Lee Silverman Voice Treatment (LSVT) approach and recruited five males and five females to participate in treatment sessions. All participants were diagnosed with Parkinson's disease at the time of recruitment. The results of one female were not analyzed because she moved to another state, midway through the study; therefore, her data was not included in the final report. This is known as: A. Bias B. Reliability C. Validity D. Attrition

D

A student study group is reviewing for an exam and learn that _______ law states that as long as the temperature and air remain constant, as the volume in an enclosed container increases, the pressure of air in that container decreases. A. Ohm's B. Pascal's C. Newton's D. Boyle's

D

A young child named Kylie was referred to an audiologist's office after she was told that she could not benefit from hearing aids because of her profound hearing loss. Her mother is extremely interested in cochlear implants and wants to know more about them. She said, "I never really thought about cochlear implants until a doctor brought that up as a possibility. I do not know much about them and need to know everything before I even consider this option for Kylie." She then gave the audiologist some information that she found online but she was not sure how accurate this information was. For example, she mentioned to the audiologist that she read that cochlear implants deliver electrical impulses to the auditory nerve, have multiple electrode arrays, have an implanted receiver, and allow the patient to discriminate between speech sounds. When reading about the potential negatives, she got overwhelmed. The audiologist made sure to stress to Kylie's mother that cochlear implants help the user perceive sound; however, she also mentioned that professionals cannot be positive as to what exactly Kylie will be able to perceive. Which of the following statements said by the mother is not true regarding cochlear implants? A. The implant is surgically implanted and delivers electrical impulses directly to the auditory nerve. B. The cochlear implant consists of a microphone, processor, external transmitter, and implanted receiver. C. All contemporary devices make use of multiple electrode arrays. D. Cochlear implants will allow the patient to discriminate between speech sounds.

D

According to Brown's stages, what is the last morpheme to be acquired by a typical child? A. prepositions B. regular third-person -s C. uncontractible auxiliary D. contractible auxiliary

D

According to Halliday, what are four of the seven functions of communicative intent that develop between 9 and 18 months of age? A. Imaginative, interactional, attribution, overextension B. Interactional, conversational, collaborative, attribution C. Collaborative, imaginative, attribution, conversational D. Heuristic, imaginative, interactional, personal

D

According to the Nyquist theorem (1928), to represent a signal faithfully, it must be sampled at a rate equal to twice its highest frequency. Brick-wall filtering works with the Nyquist theorem when analyzing data. What is the purpose of Brick-wall filtering? A. The brick-wall filter removes all of the energy below the Nyquist frequency. B. The brick-wall filter doubles all of the energy above the Nyquist frequency. C. The brick-wall filter doubles all of the energy below the Nyquist frequency. D. The brick-wall filter removes all of the energy above the Nyquist frequency.

D

All of the following are deficits of the oral phase of swallowing, except what? A. Food residue located in various places B. Aspiration before swallow C. Piecemeal swallow D. Reduced movement of the base of the tongue

D

All of the following are important for treating language disorders for patients with cerebral palsy except: A. Training parents to stimulate language at home B. Assessing the child's development periodically to determine the need for formal clinical treatment C. Implementing formal language treatment if necessary D. Not modifying the treatment procedures for language disorders to suit the individual child

D

All of the following can be used to screen for aphasia except: A.Mississippi Aphasia Screening Test-II B. Bedside Evaluation Screening Test-II C. Frequency Aphasia Screening Test-II D. Duration Aphasia Screening Test-II

D

All of the following may be expected when treating a patient with Prader-Willi syndrome, except: A. Pitch variations in voice B. Hypotonia C. Delayed receptive and expressive language skills D. Cleft lip or palate

D

All of the following may be expected when treating a patient with velocardiofacial syndrome, except: A. Intellectual disability B. Cleft palate and/or velopharyngeal insufficiency C. Language deficits D. Visual, spatial, and attentional deficits

D

An 82-year-old resident in a nursing home who suffers from Parkinson's disease is not receiving any treatment since she has been dismissed from speech-language therapy because she has met her goals. The licensed speech-language pathologist, however, plans for and provides additional therapy for the resident. This type of therapy is called: A. Speech-language therapy B. Rehabilitation therapy C. Restorative nursing D. Restorative therapy

D

An important component of a literacy assessment is evaluation of narrative schema knowledge—which is the knowledge of story structure. Which of the following strategies is least likely to be used by a speech-language pathologist when assessing narrative schema knowledge? A. Asking the client to retell a known story such as Charlotte's Web B. Reading a short story to the client and asking him multiple comprehension questions such as, "What happened in the end of the story?" C. Asking the client to make up a fake story about winning the lottery D. Assigning a book for the client to read and having him write a paragraph about his favorite character

D

An important part of treating patients with traumatic brain injuries is providing support, reassurance, information, and direction to family members. Which is most important in the middle and later stages of recovery? A. Support B. Reassurance C. Information D. Direction

D

An infant presents with tracheoesophageal fistula and esophageal atresia. The physical mentions to the speech-language pathologist that this condition probably developed between the 4th and 6th weeks of gestation. The infant exhibits significant respiratory distress, excessive oral secretions, and chronic regurgitation. The speech-language pathologist consultation with the physical should immediately recommend A. pureed foods B. thin liquids C. thick liquids D. NG-tube feedings

D

An investigator carries out a study in which the effect of rate of speech upon stuttering during sibling interaction is being investigated. The investigator gathers conversational samples from children who stutter and their siblings. In the control group, siblings are asked to speak as they normally would at home. In the experimental group, siblings are asked to speak much more quickly than they would at home. The investigator wishes to measure the effect of rate of siblings' speech upon the amount of stuttering done by the children who stutter. In other words, the investigator is asking if increased rate of siblings' speech causes children to stutter more. In this study, what is the dependent variable? A. The amount of stuttering done by children who stutter when siblings speak at a slowed rate B. The rate of speech of the siblings in the experimental group C. The rate of speech of the siblings in the control group D. The amount of stuttering done by the children who stutter when the siblings increase their rate of speech

D

Anjali, a 7-year-old, was referred for speech therapy by her classroom teacher. Anjali uses inappropriate behaviors such as loud screaming or physical violence when presented with a problem and has difficulty understanding idioms. When a confrontation with another student arises, she is unable to explain actions or feelings to the teacher. Which of the following best describes Anjali's diagnosis? A. Hearing loss B. Expressive aphasia C. Autism D. Behavior disorder

D

Apraxia of speech (AOS) can be caused by any process that damages structures involved in motor speech programming. Which of the following etiologies is the most common cause of AOS? A. Degenerative diseases B. Traumatic injury C. Left hemisphere tumor D. Vascular lesions

D

Babbling and early words have so much in common that some professionals have difficulty differentiating between the two. Which of the following statements is not a main characteristic of the transition from babbling to the first word? A. Monosyllabic utterances B. Bilabial and apical productions C. Rare use of consonant clusters D. Frequent use of glides and liquids

D

Blake, a 65-year-old man, comes to an audiologist's office and complains that he has an intense burning pain close to his ear. He states that his symptoms include vertigo and facial paralysis. He said, "I have not slept well this past week. I have been constantly feeling that my balance is off and that everything is spinning. It takes a while for the feeling to eventually stop." After sitting in the office for 10 minutes, the audiologist suddenly notices that he has small sac-like bodies on his face and neck. After further examination, the audiologist finds these same sac-like vesicles in his ear canal. To understand what Blake is experiencing, the audiologist conducts a hearing exam. Results of the testing indicate that Blake has a severe bilateral high-frequency hearing loss. The audiologist's initial impression is that Blake may have had a postnatal infection that caused cochlear damage. Which of the following specific conditions does Blake most likely have? A. Meniere disease B. Measles C. Bacterial meningitis D. Herpes zoster oticus

D

Choose the following phonological process that is not used to identify vowel errors. A. Diphthongization B. Tenseness harmony C. Decentralization D. Migration

D

Christa, a 27-year-old singer, presented with a hyperfunctional voice disorder. The clinician recommended a voice therapy technique in which words are spoken in a connected manner, with even stress, prolongation of sounds, and soft glottal attack. This therapy technique is recommended for clients with hyper-functional voice: A. Open-mouth approach B. Masking C. Glottal attack changes D. Chant-talk method

D

Christa, a 27-year-old singer, presented with a hyperfunctional voice disorder. The clinician recommended a voice therapy technique in which words are spoken in a connected manner, with even stress, prolongation of sounds, and soft glottal attack. This therapy technique is recommended for clients with hyperfunctional voice: A. Open-mouth approach B. Masking C. Glottal attack changes D. Chant-talk method

D

Different aphasia types share common features of communication impairments. Therefore, a clinician who is interested in making a typological diagnosis needs to fully understand the unique features of each type. Among the following statements, which correctly helps distinguish the two types contrasted? A. Auditory comprehension in patients with conduction aphasia is poor, whereas it is excellent in patients with transcortical sensory aphasia. B. Mixed transcortical is the most severe form of aphasia, whereas global aphasia affects only word output. C. Grammatical structures are impaired in patients with Wernicke's aphasia, whereas they are intact in patients with Broca's aphasia. D. Transcortical sensory aphasia patients have fluent speech, normal prosody, and good articulation, whereas transcortical motor aphasia patients have nonfluent, paraphasic, and agrammatic speech.

D

Even though there is limited to no evidence on its effectiveness, what is common form of treatment for comatose or semi-comatose patients? A. Orientation training B. Medications C. Behavioral training D. Sensory stimulation

D

HIPPA's Privacy Rule requires organizations to designate a privacy officer to: A. establish measures for obtaining a patient's written consent to use or disclose his or her health information for purposes alternative to treatment and payment. B. improve or amend policies to uphold a patient's rights under HIPPA. C. inform a patient about HIPPA. D. manage all activities related to HIPPA.

D

In preparing for feeding and swallowing with a patient with a tracheostomy tube, which of the following is recommended while conducting therapy? A. Have the patient sit in the bed at a 120 degree angle before occluding the patient's tracheostomy B. Use cervical auscultation to determine if there is any aspiration C. There is no need to occlude the tracheostomy during and immediately after the swallow, as there is no chance of aspiration D. Occlude the patients tracheostomy during and immediately after the swallow, as the exhalatory airflow after the swallow may contribute to clearance of residual food from the top of the airway, reducing the change of aspiration after the swallow.

D

In preschoolers who stutter, there appears to be sex-specific differences in _____________ between preschool boys and girls who stutter. A. increased amplitudes of articulatory movement B. increased velocities of articulatory movement C. patterns of articulatory coordination that are less variable D. speech motor control processes

D

In treating the communication deficits of a young adult with traumatic brain injury, you would do which of the following? A.Refrain from using techniques to increase orientation and attention because the inappropriateness of these treatment targets subside in due course. B. Exclusively use cognitive rehabilitation. C. Introduce a variety of treatment activities to promote faster recovery. D. Withhold attention from irrelevant and inappropriate responses.

D

Jack is an 8-year-old boy who is a chronic mouth breather. He attends an evaluation with his mother, who mentions that the ENT informed her that in young children, respiration could be obstructed when the pharyngeal tonsils are inflamed or swollen. You concur with the ENT and mention that it is possible that the root cause of her son's chronic mouth breathing is swelling of the: A. Inferior constrictor of the pharynx B. Trachea C. Nasal conchae D. Tonsillar ring

D

Jaxon, a 78-year-old male, recently experienced a cerebrovascular accident leading to damage of the border areas between the tissues that are supplied by the anterior, middle, and posterior cerebral arteries. His speech is nonfluent, although his repetition of words and sentences is preserved. All other aspects of Jaxon's language are severely impaired. Jaxon most likely presents with: A. Global aphasia B. Transcortical motor aphasia C. Transcortical sensory aphasia D. Mixed transcortical aphasia

D

Laryngeal cancer patients who are treated with radiotherapy for a small tumor may experience all of the following except: A. Temporary hoarseness B. Temporary vocal roughness C. Small changes in saliva flow D. Inability to phonate

D

Lisa, a 19-year-old college student, was in a car accident and was airlifted to the trauma center at a local hospital. The paramedics at the accident scene had to perform an emergency intubation to permit her to breathe. A week after the accident, Lisa was discharged from the hospital and was breathing normally. A month later, she returned to the hospital complaining of hoarseness and breathiness. The laryngologist performed an evaluation and noticed that a unilateral localized inflammatory vascular lesion had developed on the vocal process of her arytenoid cartilage. The laryngologist believed that the intubation may have caused A. hyperkeratosis. B. leukoplakia. C. hemangioma. D. a granuloma

D

Peter has suffered a cerebrovascular accident (CVA); the neurologist reports lesions in the third convolution of the left cerebral hemisphere. Based on this, you conclude that the damaged area is A. the occipital lobe. B. the basal ganglia. C. Wernicke's area. D. Broca's area

D

Peter is a young client who you normally treat for delayed language; however, recently, he has been demonstrating dysfluencies during conversation. A lot of clinicians agree that typically developing children experience a period of dysfluent speech during the preschool years. During therapy, you decide to monitor Peter's speech for the types of dysfluencies he displays. Which of the following is a type of dysfluency that is not typical for a child experiencing normal developmental dysfluencies? A. Whole word repetitions B. Phrase repetitions C. Syllable interjections D. Final word repetitions

D

Public Law 94-142, the education for all Handicapped Children Act of 1975, was later reauthorized and retiled as A. the Americans with Disabilities Act B. the Education of Disabled Individuals Act C. the Handicapped Individuals Education Act D. the Individuals with Disabilities Education Act

D

Recognition is very important in caregiver-infant attachment because it is the ability of two individuals to attend to and recognize each other's presence, as distinguished from that of others. Before therapy begins, it is important to recognize an infant's signals for recognition. Infants signal recognition through all of the following except: A. Sucking B. Crying C. Calmness D. Fixation

D

Research has indicated that in a longitudinal study of 36 children who stuttered and 34 controls, Garnett et al. (2018) found that in persistent development stuttering, there was strong evidence of a primary deficit in the left hemisphere in the ________________ A. posterior parietal cortex. B. primary somatosensory cortex. C. supplementary motor area and the secondary motor cortex. D. lateral premotor cortex and primary motor cortex

D

Sara has arterial damage that causes her to have cognitive deficits such as impaired judgment, problems concentrating, and difficulties with reasoning. According to the surgeon, damage to the affected artery can also cause a person to have paralysis of the feet and legs. Damage to which artery produces these effects? A. Internal carotid B. External carotid C. Middle cerebral D. Anterior cerebral

D

Senbo is a third-grade student who speaks Afrikaans. Her teacher refers her for assessment because she is having academic difficulty. To evaluate Senbo's language skills, you use dynamic assessment, an alternative approach in which the clinician A. collects various kinds of work samples from the child being evaluated. B. compares a child's performance with the established norms. C. seeks to sample the child's speech and language in naturalistic settings. D. evaluates her over time in a test-teach-retest format.

D

Sound is defined as vibrations or disturbances of molecules in a medium. This means that A. the sound waves actually travel from point A to point B. B. sound waves are always audible; otherwise, we could not study them. C. a rigid medium can be as good as a flexible medium in transmitting sound. D. sound waves are disturbances in adjacent molecules within a medium.

D

Speech generating devices can be set to use a voice reflective of a user's age and gender. Which type of speech output uses recorded speech that is stored on an augmentative and alternative communication (AAC) device and allows individuals to produce messages using appropriate inflection and intonation? A. Dynamic speech B. Preference speech C. Synthesized speech D. Digitized speech

D

Speech reception thresholds (SRTs) are A. determine by the patient's response to a list of monosyllabic words presented at a low level of hearing B. determine by looking at the patient's pure-tone test results at the frequencies most important to speech C. the lowest levels of hearing at which a person can understand 100% of the words presented D. the lowest levels of hearing at which a person can understand 50% of the words presented

D

Speech-language pathologists have recently become members of the team that serves clients with developmental reading and writing disabilities. A 6-year-old child comes into your office because his teacher believes that he may have literacy problems. You evaluate his reading and writing skills and take note of several behaviors. Which of the following does not correspond with your client's age? A. When he was reading, simple words were sounded out. B. The subject matter of his writing typically expressed feelings and memories. C. He associated sounds with alphabetic symbols. D. He is able to write sentences that are longer than spoken sentences.

D

Standardized tests are limited in their usefulness because A. they do not allow for comparative evaluation of performance. B. they draw nationally representative samples that may not represent local samples. C. they are not always accepted as a basis to determine service eligibility in schools. D. they sample participants (children) and responses in a limited manner.

D

The ______________ research design is the after-the-facts research. It involves looking for events that have already occurred. Researchers begin by defining the effect as it currently exists. They then look back to describe potential causes of the events. A. single subject B. comparative C. developmental D. retrospective

D

The ability of a child to modify her understanding of a changing environment often becomes evident during early language development. The ability of a child to exhibit assimilation and accommodation when presented with a new environment are features of: A. Equilibrium B. Imitation C. Organization D. Adaptation

D

The function of the __________ is to allow optimum contraction and expansion of the lungs during breathing. A. mediastinum B. bronchioles C. alveoli D. pleura

D

The parents of 4-year-old Mariah consult a clinician. They have moved several times since Mariah was born; her father is a construction worker, and her mother works full-time inside the home caring for Mariah and her three siblings. The family has "struggled financially," according to Mariah's mother, Mrs. E. The clinician obtains the following information from Mrs. E. about Mariah's history: Mariah was born with a cleft of the soft palate and "a funny-looking face." Mrs. E. shares that "it was a nightmare to feed Mariah when she was a baby, the milk always came out through her nose." Mrs. E. begins to get angry as she describes the hospital staff in the town where Mariah was born. She states, "They said Mariah had some sort of syndrome and that she might always have special needs. I think that's bunk. Those people were so insensitive. They were just too busy to work with her properly. Mariah will be fine. I know she has had her problems, but I'm going to put her into a Montessori preschool because she is so smart." When the clinician sees Mariah for the first time, she notes that Mariah has a wide nose, small ears, almond-shaped eyes, and an elongated face. In addition, the clinician finds during assessment that Mariah has a significant expressive language delay. Mariah's mother is exemplifying the defense mechanism of: A. Projection B. Reaction formation C. Displacement D. Denial

D

The physical characteristics of Moebius syndrome may include involvement of any of the following cranial nerves, except the: A. Facial nerve B. Hypoglossal nerve C. Trigeminal nerve D. Vagus nerve

D

The reduction in density of an item is called: A. reflection B. refraction C. reverberation D. rarefaction

D

The speech-language pathologist in a school is trying out a new augmentative and alternative communication (AAC) device for a child and believes that facilitator training is important for the caregiver to do all the following except: A. Accurately use the device B. Troubleshoot the device C. Add/take away symbols, buttons, or folders D. Reduce the independence required by the child

D

The term coarticulation refers to which of the following? A. Speech sounds being modified due to the influence of adjacent sounds to the point that there are perceptible changes in the speech sounds B. The extent to which vocal tract configuration changes shape during the production of consonants and vowels in running speech C. The influence of various syllables upon one another when a client recites a phonetically balanced list of words D. The influence of one phoneme upon another in production and perception, wherein two different articulators move simultaneously to produce two different speech sounds

D

The transverse band of tissue on either side of the tongue is (are): A. Central sulcus B. Lingual frenulum C. Sublingual glands D. Sublingual folds

D

This is an inherited degenerative disease that is associated with spasticity, lower motor neuron weakness, and difficulty coordinating voluntary motor movements: A. Multiple sclerosis B. Shy-Drager syndrome C. Progressive supernuclear palsy D. Freidrich's ataxia

D

To evaluate the effects of a phonological treatment procedure, an investigator started by establishing the baselines of target phoneme productions, then offered treatment to all children who were base rated, withdrew treatment for a period of time, and finally offered the treatment again. This is an example of A. a pretest-posttest control group design. B. a multiple-baseline design. C. a case study. D. a ABAB design

D

Tumors located on the free margin of one vocal fold with only minimal extension are usually treated with a laryngectomy known as all of the following except: A. Hemilaryngectomy B. Vertical laryngectomy C. Extended hemilaryngectomy D. Horizontal supraglottic laryngectomy

D

What are cartilages that are cone shaped and are located at the posterior of the aryepiglottic folds and are occasionally fused with the arytenoid cartilages? A. Cricoids B. Laminas C. Cuneiforms D. Corniculates

D

What determines the intensity at each harmonic in vowel production? A. Nothing B. Respiration C. Vocal folds D. Vocal tract

D

What disability is not recognized under the Individuals with Disability Education Act? A. Autism spectrum disorder B. Deafness C. Emotional disturbance D. Language learning disabilities

D

What is a pink or white wart-like growth that can be found anywhere in the airway and make a person's voice sound hoarse, breathy, and low pitched? A. Hyperkeratosis B. Leukoplakia C. Hemangioma D. Papilloma

D

What is an important factor to consider when treating a voice disorder for a patient with cerebral palsy? A. Understanding the effects of how the voice disorder can improve limb movement B. Recognizing how improvement of the voice disorder can increase mandibular range of motion C. Considering the implications that the voice disorder may have on brain stimulation D. Being aware that the voice disorder may be attributable to respiratory problems associated with cerebral palsy

D

What is true about Broca's aphasia? A. Speech is generally meaningless. B. Grammar is typically intact. C. It is never associated with apraxia of speech or dysarthria. D. It is often, though not always, caused by damage to the posterior inferior frontal gyrus in the left hemisphere.

D

What specifically prohibits practitioners from charging for future services or incomplete services? A. Joint Commission on Accreditation of Health Care Organizations (JCAHO) B. Health Insurance Portability and Accountability Act (HIPPA) C. Local Coverages Determinations (LCD) D. The List of Excluded Individuals and Entities (LEIE)

D

When analyzing a client's speech sample with an unrepaired cleft palate, what determines the timing of speech intervention? A. Intelligibility B. Misarticulations C. Nasal emission D. Surgical procedures

D

When apraxia of speech is characterized by limited vocalizations, there are some techniques that may be successful during speech therapy. Which of the following is not a technique that can elicit vocalizations in patients with apraxia who are mute? A. Singing B. Automatic speech tasks C. Pushing on the abdomen D. Self-learning

D

When assessing and evaluating language disorders, there are several broad diagnostic categories that provide a framework for understanding language disorders. Even though deficient language is not the only clinically significant feature, of the following conditions which category does not relate to language disorders? A. Specific language impairment (SLI) B. Autism spectrum disorder (ASD) C. Phonologic disorders D. Childhood apraxia of speech (CAS)

D

When evaluating a child with cerebral palsy, the following symptom is often noticed: A. Increased rate of speech B. Equal stress is produced on all syllables C. Fatigue is not an issue D. Resonance difficulties

D

When treating a client who stutters, a clinician may ask the client to repeat back everything the clinician reads from a book. This technique is called what? A. Prolonged speech B. Delayed auditory feedback C. Rate reduction D. Shadowing

D

Which goal of stuttering modification therapy is incorrect: A. Reducing the severity of stuttering. B. Making stuttering less abnormal. C. Modifying the shape of stuttering. D. Changing the frequency of stuttering.

D

Which of the following does not typically occur in children with cerebral palsy? A. Impaired language development B. Learning difficulties C. Academic problems D. Apraxia of speech

D

Which of the following is considered the single most important structure for the production of speech, and is controlled by a series of intrinsic and extrinsic muscles? A. Lungs B. Occipital lobe C. Velum D. Tongue

D

Which of the following is not a major characteristic of hyperkinesias? A. Spasm B. Hemiballism C. Dystonia D. Bradykinesia

D

Which of the following is the most common etiology of hypokinetic dysarthria? A. Closed head injury B. Nonhemorrhagic stroke C. Extrapyramidal disorder D. Parkinson's disease

D

Which of the following is the most common form of mixed cerebral palsy? A. Dyskinetic-athetoid B. Ataxic-dyskinetic C. Spastic-ataxic D. Spastic-dyskinetic

D

Which of the following muscles helps adduct the vocal folds? A. Hyoglossus B. Cricothyroid C. Posterior cricoarytenoids D. Transverse arytenoids

D

Which one of the following is not a feature of norm-referenced, standardized tests? A. They prescribe systematic procedures for administration and scoring of the test. B. They allow for a comparison of a client's score to that of a normative sample. C. They allow for consistency of administration and scoring across examiners. D. They generate information that can be used to create treatment goals and assess treatment progress.

D

Which one of the following statements is false? A. Non-iconic symbols are geometric, abstract, and arbitrary and must be specifically taught. B. The Picture Exchange Communication System (PECS) is effective. C. A hieroglyphic picture of a house is an iconic symbol indicating the word house. D. In direct selection, the user is offered available messages by a mechanical device or communication partner; the messages are offered sequentially until the AAC user indicates the messages he or she wants to communicate

D

Which type of reinforcer is exemplified by verbal praise, attention, and facial expressions? A. Primary reinforcer B. Secondary reinforcer C. Unconditioned reinforcer D. Social reinforcer

D

You are assessing a 60-year-old woman for a possible diagnosis of aphasia. You are interested finding out the specific type of aphasia that the woman is experiencing. You notice that she has a inability to whistle, cough, or wink on command. This condition alone might suggest which type of aphasia that the woman has? A. Wernicke's aphasia B. Anomic aphasia C. Occulomotor apraxia D. Buccofacial apraxia

D

You are developing a treatment plan for a 30-year-old man who sustained traumatic brain injury in an auto accident. Premorbidly, this man had excellent language skills; he had a graduate degree in linguistics and was a college lecturer. Among several others, which set of goals and procedures would you select in treating this person? A. I would give polite and appropriate responses to his irrelevant and tangential questions and then ask him not to ask such questions during therapy. B. I would teach him to guess what others mean when he does not fully understand what they say and respond based on his best guess. C. I would teach narrative skills by telling him complex stories from the beginning because of his higher education and advanced premorbid language skills. D. I would give such signals as "listen carefully," "I am going to say something different now," and "pay attention to what I am about to say" to improve attention to communication.

D

You are evaluating a 10-year-old boy who was referred because of difficulties associated with partial submucous cleft palate accompanied by a bifid uvula. He is being teased at school for "sounding funny," and his parents are concerned about how he talks. During your evaluation, what can you probably expect to find? A. Hypernasality, leading to difficulty producing nasals adequately B. Hyponasality, accompanied by increased intraoral breath pressure, leading to difficulties with adequate production of liquids and glides C. Intermittent hyponasality, leading to difficulty producing nasals adequately D. Hypernasality, accompanied by decreased intraoral breath pressure, leading to difficulties with adequate production of fricatives, affricates, and plosives

D

You are evaluating a 5-year-old boy whose mother has a history of alcohol abuse while she was carrying her son. In your assessment, you would look for specific speech and language problems; in addition, you would look for which of the following positive or negative signs? A. Normal motor and intellectual development, normal play activities, and normal facial and skull features B. Language problems, but no speech problems, coupled with good eye contact and generally compliant behavior C. Lack of gestures, good eye contact, and lack of attachment to new people D. Low birth weight and length, behavior problems, and possible swallowing difficulties

D

You are performing a diagnostic evaluation on Belle, a 4-year-old girl, who was brought to the clinic by her mother due to concerns about her daughter's speech. During the evaluation, you note that Belle often produces /t/ instead of /k/ in the initial position of words. She also produced /w/ for /l/ in the word, leaf, and /d/ for /dÊ'/ in the word jump. Which of the following is an active phonological process that is not appropriate for Belle's age and should be addressed during therapy? A. Gliding B. Final consonant deletion C. Stopping /dÊ'/ D. Fronting

D

You are planning treatment for a child who needs to learn morphological features of language. You have selected four grammatical morphemes to teach at the word level. You will then teach each word to a training criterion. You show a stimulus card, ask a question, wait for the response, reinforce the correct response, score the response, and wait for a few seconds before presenting the next opportunity for the child to produce the target morpheme. What is this procedure called? A. The shaping method B. The successive approximation C. The naturalistic method D. The discrete trial method

D

You are working in a hospital, and a 72-year-old patient, Grace, is referred to you. She has difficulty paying attention to a conversation, staying on topic, remembering information, responding accurately, and following directions. She most likely has: A. Apraxia of speech B. Wernicke's aphasia C. Transcortical motor aphasia D. Cognitive communication disorder

D

You are working in a public school, and a distraught parent calls you. She states that her son who was prenatally exposed to drugs has been denied special education services including speech-language intervention. Why may children who are prenatally exposed to drugs be denied services in public schools? A. Their language problems are not severe enough to qualify. B. They do not have other kinds of problems associated with disorders of communication. C. They are classified as having learning disabilities, not language impairments. D. Their language problems are not readily detected by standardized language measures.

D

You are working with a 7-year-old Spanish-speaking girl, Rosa, who is in the process of learning English as a second language. Which of the following would not be typical for her in terms of predictable productions based on Spanish influence? A. Insertion of schwa before word-initial /s/ clusters B. Devoicing of final consonants C. b/v substitutions D. v/f substitutions in medial position of words

D

You are working with a 7-year-old Spanish-speaking girl, Rosa, who is in the process of learning English as a second language. Which of the following would not be typical for her in terms of predictable productions based on Spanish influence? A.Insertion of schwa before word-initial /s/ clusters B. Devoicing of final consonants C. b/v substitutions D. v/f substitutions in medial position of words

D

You are working with a 7-year-old child for language therapy, and you are teaching the accurate production of plural -s in words. You record the correct and incorrect responses on each attempt you ask the child to make. This practice is known as A. the pretest-postest method B. intermixed probing C. the shaping method D. the discrete trial procedure

D

You have been asked to counsel with John, a 70-year-old man who has smoked and drank alcohol since he was a teenage. He now has laryngeal cancer, and, before surgery, the surgeon asks you to talk to John about esophageal speech. You explain to John that there are two basic types of esophageal speech. In one method, the patient is taught to keep the esophagus open and relax while inhaling rapidly. In the other method, the patient impounds the air in the oral cavity, pushes it back into the esophagus, and vibrates the cricopharyngeus muscle. What is the second method called? A. inhalation method B. laryngeal airway resistance method C. inhalatory injection method D. injection method

D

_______________ states that all children and youth with disabilities from ages 3 to 21 years are guaranteed free and appropriate public education in the "least restrictive environment," including special education and related services. A. P.L. 142-204 B. P.L. 29-204 C. P.L. 49-142 D. P.L. 94-142

D

The corpus striatum is composed of three nuclear masses, which are the

Globus pallidus, caudate nucleus, and putamen

Which muscles from the list below are the most involved in adducting the vocal folds?

Lateral cricoarytenoids and transverse arytenoids

Which branch of the vagus nerve (cranial nerve X) innervates the cricothyroid muscle?

Superior laryngeal nerve

Damage to the sensory component of Cranial Nerve V would result in loss of tactile sensation to the: ______________ ________________ of the tongue

anterior two thirds

These are composed of a ring of connective tissue and muscle extending from the tips of the arytenoid cartilages to the epiglottis. They separate the laryngeal vestibule form the pharynx and help preserve the airway.

aryepiglottic folds

The structure that regulates body posture, equilibrium, and coordinated fine motor movements is the

cerebellum

An important structure adjacent to the brainstem that contains the hypothalamus (which controls emotions) and the thalamus (which relays sensory impulses to various portions of the cerebral cortex) is called the

diencephalon

The neurons that transmit information away from the brain are called

efferent neurons

When a person is producing voiced and voiceless /th/, the muscle that is most involved is the

genioglossus

The structure at the inferior portion of the tongue that connects the tongue with the mandible is called the

lingual frenum

The primary muscle of the lips is the

orbicularis oris

Muscles that contribute to the velopharyngeal closure through tensing or elevating the velum are the

palatoglossus, tensor veli palatini, and levator veli palatini


Conjuntos de estudio relacionados

Biology Chapter 49: Osmoregulation

View Set

Anatomy of Abdomen Part 1- Peritoneum

View Set

Health and Society Exam 2 Review

View Set

Comparing Poetry: Poetic Devices

View Set

BYU APUSH/APHIST Part 1 All Questions

View Set

IBM (Chapter 4 Connect Assignment 4)

View Set